0% found this document useful (0 votes)
10 views56 pages

Recalls 1

The document discusses the evolution of public health nursing in the Philippines, highlighting its alignment with the Department of Health and the Millennium Development Goals (MDGs). It outlines the determinants of health, essential health services, and various health theories and models relevant to public health practice. Additionally, it emphasizes the importance of community health diagnosis and the role of nurses in promoting health and preventing disease through organized community efforts.

Uploaded by

jrsteve122001
Copyright
© © All Rights Reserved
We take content rights seriously. If you suspect this is your content, claim it here.
Available Formats
Download as PDF, TXT or read online on Scribd
0% found this document useful (0 votes)
10 views56 pages

Recalls 1

The document discusses the evolution of public health nursing in the Philippines, highlighting its alignment with the Department of Health and the Millennium Development Goals (MDGs). It outlines the determinants of health, essential health services, and various health theories and models relevant to public health practice. Additionally, it emphasizes the importance of community health diagnosis and the role of nurses in promoting health and preventing disease through organized community efforts.

Uploaded by

jrsteve122001
Copyright
© © All Rights Reserved
We take content rights seriously. If you suspect this is your content, claim it here.
Available Formats
Download as PDF, TXT or read online on Scribd
You are on page 1/ 56

COMPREHENSIVE PHASE - RECALLS 1 (NP1) C.

organization of medical and nursing services for the early diagnosis and
Situation: Public Health Nursing in the Philippines evolved alongside the institutional preventive treatment of disease
development of the Department of Health, the government agency mandated to D. education of the individual in personal hygiene
protect and promote people's health and the biggest employer of health workers 4. Public health nursing is strongly built on theories of health promotion and disease
including public health nurses. prevention. This model found that information alone is rarely enough to motivate
1. In response to above trends, the global community, represented by the Nations people to act for their health. Individuals must know what to do and how to do it before
General Assembly, decided to adopt a common vision of poverty reduction and they can take action.
sustainable development in September 2000. This vision is exemplified by the A. Health Belief Model – all related to the client’s perceptions. Individuals must
Millennium Development Goals (MDGs) which are based on the fundamental values know what to do and how to do it before they can act.
of freedom, equality, solidarity, tolerance, health, respect for nature, and shared B. Health Promotion Model – depicts complex multidimensional factors which
responsibility. Which among these is not a Millennium Development Goal? people interact with as they work to achieve optimum health
A. Improve maternal health C. Social Learning theory – it is based on the belief that learning takes place in a
B. Combat HIV/AIDS, malaria and other diseases social context; people learn from one another and learning is promoted by
C. Ensure environmental sustainability modeling or observing other people – copy one another
D. Promote gender equity and empower women - should be equality D. General Systems Theory – the theory was analyzing interrelationships of the
2. Factors or things that make people healthy or not, known as determinants of health elements within the client and the environment. It is composed of outputs,
are listed by the World Health Organization to include all except one: feedback, subsystem, and suprasystems.
A. Genetics 5. It is based on the assumption that behavior change takes place over time, and
B. Gender progresses through stages. Each stage is stable and is open to change; meaning one
C. Location may stop in one stage, progress to the next stage or return to a previous stage.
D. Culture A. General Systems Theory
DETERMINANTS OF HEALTH B. Transtheoretical Model – different steps of change; not linear; one may stop in one
1. Income and social status stage, progress to the next stage or return to a previous stage
2. Education C. PRECEDE-PROCEED Model – for community diagnosis
3. Physical environment ✓ It provides a model for community assessment, health education planning, and
4. Employment and working conditions evaluation.
5. Social support networks ✓ PRECEDE, which stands for predisposing, reinforcing, and enabling constructs in
6. Culture educational diagnosis and evaluation is used for community diagnosis.
7. Genetics ✓ PROCEED, stands for policy, regulatory, and organizational constructs in
8. Personal behavior and coping skills education and environmental development, is a model for implementing and
9. Health services evaluating health programs based on PRECEDE.
10. Gender D. Milo’s Framework for Prevention It includes economic, political and environmental
3. According to C. E. Winslow, “Public health is the science and art of (1) preventing health determinants rather than just the individual's perceptions. This theory
disease, (2) prolonging life, and (3) promoting health and efficiency through organized encourages the nurse to understand health behaviors in the context of their societal
community effort.” Which is considered as the key phrase definition of public health? milieu.
A. development of the social machinery to ensure everyone a standard of living Situation: WHO defines PHC as essential health care made universally accessible to
adequate for the maintenance of health individuals and families in the community by means acceptable to them through their
B. through organized community effort – it has to be everyone full participation and at a cost that the community and country can afford at every stage
of development.
6. What is considered the legal basis of PHC in the Philippines? B. Intersectoral linkages – 2 or more other sectors
A. LOI 949 – legal basis of PHC C. Interfacility linkages
B. LOI 497 D. Intra-facility linkages
C. LOI 789 Situation: King Kong is learning to use the available resources in Brgy. Kagalingan in
D. LOI 747 providing care to his patients. He firmly believes in the efficiency in using traditional
7. This is signed by which President of the Philippines? and alternative medicine in his plan of care so that it will be easier for the people to
A. Pres. Ferdinand Marcos - accept and follow.
B. Pres. Gloria Aquino 11. A 7-year-old kid is being accompanied by his mother to the clinic. The child has
C. Pres. Ramon Magsaysay been experiencing abdominal pain, and is presenting with a round belly. His mother
D. Pres. Ferdinand Marcos Jr. said that he did drink any “pampurga” when he was young. What medicinal plant
8. The PHC has 8 essential health services in which it aims to improve. All of these should he give?
are included, except: A. Lagundi – cough
A. Maternal and Child health B. Yerba Buena – pain
B. Nutrition C. Tsaang Gubat – stomachache
C. Exclusive Program for Immunization - Expanded D. Niyog-niyogan – pampurga
D. Education for health 12. How would King Kong tell the mother to prepare and administer the medicinal
8 ESSENTIAL HEALTH SERVICES plant?
E – Education for health A. Decoction
L – Locally endemic diseases B. Leaves are eaten
E – Essential Drugs C. Poultice
M – Maternal and Child Health D. Seeds are used
E – EPI 13. People at Brgy. Kagalingan often complains having pain in their joints after eating
N – Nutrition so much internal organs and beans. What can he advise to the people to take?
T – Treatment for communicable and non-communicable diseases A. Sambong – edema
S – safe sanitation B. Bayabas – antiseptic
9. Considerations are made regarding the health services being provided to the C. Ulasimang bato – gout = high in uric acid
people. These have to be to be available acceptable, affordable and accessible to the D. Akapulco – antifungal
people to reach the success of Primary Health Care. Which criteria questions whether 14. This medicinal plant is used to help lower hypertension and blood cholesterol.
the health service is offered in health care facilities or is provided on a regular and A. Ampalaya – DM type 2; increases insultin sensitivity
organized manner? B. Bawang – lower hypertension
A. Acceptability – service is compatible with culture and traditions of the C. Akapulko – antifungal
population D. Sambong – edema
B. Affordability – afford services 15. Type II diabetes is also rampant in the barangay. He conducts group health
C. Accessibility – reached within 30 minutes teachings to include which plant into their diet?
D. Availability A. Ampalaya – DM type 2; increases insultin sensitivity
10. The PHC uses multisectoral approaches to communicate, collaborate, and B. Bawang – lower hypertension
convene to achieve universal health care. When the doctor has to refer the victim of C. Akapulco – antifungal
VAW to DSWD, what referrals or linkages will they use? D. Sambong – edema
A. Intra-sectoral linkages – with DOH
Situation: The Centers for Disease Control and Prevention (CDC) established an office 21. According to Freeman and Heinrich, community health diagnosis is based on three
in the Philippines in 2022 to strengthen and expand existing partnerships with the interdependent, interacting and constantly changing conditions. Which of these is not
Philippines Department of Health (DOH). included?
16. These vital statistics indicate the natural decrease in population. A. The health status of the community, including the population’s level of
A. Crude birth rate – natural increase vulnerability (+)
B. Crude death rate – natural decrease B. Diversity within the community, focusing on making comprehensive and
C. Prevalence Rate – old and new cases inclusive community health care
D. Incidence rate – new cases C. Community health capability or the ability of the community to deal with its
17. Nurse Caloy wants to determine the health status of a population and infer their health problems (+)
longevity of life and lifestyle by using statistics. What vital statistics shall he use? D. Community action potential, or the patterns in which the community is likely to
A. Prevalence Rate work on its health problems (+)
B. Swaroop’s Index – no. of people in certain age 22. Collecting data is important in community assessment. There are different
C. Maternal Pregnancy Rate methods as to how one can obtain data. Which method is used to elicit and explore
D. Case Fatality Ratio – index of killing power of disease opinions of people, determine their attitudes and practices regarding limited set of
18. These statistics indicate the health status of the community. concepts? Participants selected should have characteristics that will be common to
A. Fetal death rate - them, and characteristics that will also differentiate them from one another.
B. Neonatal mortality rate A. Group interviews
C. Infant mortality rate – most vulnerable B. Key Informant Interviews – with expert/person with authority
D. Mortality rate C. Focus Group Discussions -
19. This is also known as the index of pregnancy wastage. D. Windshield Survey – rapid appraisal method; ocular survey
A. Fetal death rate 23. You wanted to be as comprehensive as possible in identifying different problems
B. Neonatal mortality rate in the community. You have identified that the RHU is having difficulty in providing
C. Infant mortality rate services to their population coverage. Further assessment shows that there are a lot
D. Mortality rate of Plantilla positions left unfilled. What kind of problem is this?
20. Retrieving this number, this will help direct the efforts to improve maternal A. Health status problem – vital statistics
postpartum care in the community. This also shows that an improvement in B. Health resources problem – manpower equipment, money, materials
environmental factors must be promoted to improve the numbers. C. Health-related problem – social, economic, political, and environmental
A. Fetal death rate D. Health concern problem
B. Neonatal mortality rate – <28 days 24. You have also observed that the local government does not make health as their
C. Infant mortality rate priority thus leaving their BHCs and RHUs lacking of resources and manpower. There
D. Mortality rate is also a lack health program implementation as no support is received. This is
Situation: In Community Health Nursing, the community is not simply a context of the classified as:
existence of the nurse’s clients nor is it just a setting for our nursing interventions. A. Health status problem
Spradley (1990) emphasizes that the community is the primary client for two main B. Health resources problem
reasons. First, the community has a direct influence on the health of the individual, C. Health-related problem – talks about political side
families, and sub-populations. Second, it is the level that most health service provision D. Health concern problem
occurs. 25. Reviewing the records, you found out that there is an increasing number of non-
communicable diseases in the barangay. You classify this problem as?
A. Health status problem
B. Health resources problem D. Outcome evaluation – long term goals = goal of the program
C. Health-related problem PALMR
D. Health concern problem ✓ S – structure = materials, equipment, staffing
Situation: Nurse Jessa attempts to identify the possible factors associated with the o Done by the management
increasing cases of malnutrition in Brgy. Hintuturo. ✓ P – process = plans and procedures = staff
26. She attempts to derive the risk of an individual developing a disease due to a ✓ O – outcome = results
particular exposure by comparing the occurrence of a disease in a population exposed 30. In evaluating programs, we use indicators to identify which factors should be
to the suspected factor. She wants to use an estimate which is the direct measure of achieved and resolved by the community. Using the data information gathered from
strength of association between a suspected cause and effect. Her existing data the indicators can be used as basis for recommendations on the future and direction
includes the incidence rates of those who had who were exposed and incidence rate of health programs to be evaluated. Which indicator shows to what extent something
among those who are not exposed. Which risk estimate shall she use? that has been made available is actually being used for that purpose?
A. Odds Ratio – probability of disease development as a result of being exposed A. Relevance
to suspected factor B. Coverage
B. Attributable Risk – absolute effect of the exposure of the excess risk of disease C. Efficiency
to a causal agent D. Utilization
C. Incidental Risk Ratio Indicator Purpose
D. Relative Risk Availability Show whether something exists and if it is available
27. Using the PRECEDE- PROCEED model, she is also aiming to identify the Relevance Show how relevant or appropriate something is
contributing factors of a problem that may become the focus of subsequent Accessibility Determine whether what exists is actually within reach of
intervention. These are characteristics of the client that motivates behavior related to those who need them
health, and it can be described in terms of client’s knowledge, attitudes, values and Utilization Show what extent something that has been made available
perceptions. is actually being used for that purpose
A. Predisposing factors Coverage Show what proportion of those who need something are
B. Internal factors actually receiving it
Quality Show the quality or standard of something
C. Enabling factors – environmental characteristics that enables health behaviors
Effort Show how much and what is being invested in order to
D. Reinforcing factors – reward/punishment as a consequence to behavior
achieve the objectives
28. These are considered the factors which can be rewards or punishments following
Efficiency Show whether resources andactivities are being put to use
or anticipated as a consequence of a health behavior. to achieve the objectives
A. Predisposing factors Impact Show if what you are doing is really making any difference
B. Internal factors Situation: Nurse Dan is handling patients with respiratory problems. Tommy has been
C. Enabling factors diagnosed with ineffective airway clearance due to excessive secretions and is at risk
D. Reinforcing factors of infection from these retained secretions. As part of Nurse Dan’s care plan, the goal
29. As the program comes to an end, she plans to look into the three aspects of the is to loosen and remove these excessive secretions from Tommy’s airway.
program. She wants to measure the immediate effects of the program and determine 31. After listening to Tommy’s lung sounds, Nurse Dan identifies congestion in the
whether the objectives of the program were met. What evaluation measure will she upper lobes. To drain the anterior and posterior apical segments of the lungs during
use? percussion, the correct position would be:
A. Process evaluation – activities of the program, quality, and reach A. Client lying on his back, then flat on his abdomen in the Trendelenburg position
B. Impact evaluation – short term objectives = talks about immediate effects B. Client seated upright in bed or on a chair, then leaning forward while sitting,
C. Structure evaluation then flat on his back and abdomen
C. Client lying flat on his back, then flat on his abdomen C. Remove the medication patch, wipe the area dry, and place the electrode pad
D. Client lying on his right side, then left side in the Trendelenburg position in correct position – med patch = decrease effectiveness of shock
32. When documenting the results of Tommy’s treatment, Nurse Dan should include D. Place electrode pad on the victim’s right abdomen
the following details EXCEPT: 37. It is important for her to recognize which rhythms are for cardioversion, defibrillation
A. Color, amount, and consistency of sputum or non-shockable. Which among the following is non-shockable?
B. Character of breath sounds and respiratory rate before and after the procedure A. Ventricular fibrillation
C. Amount of fluid intake before and after the procedure B. Ventricular tachycardia
D. Significant changes in vital signs C. Asystole – no rhythm to reset + Pulseless Electrical Activity
33. When evaluating Tommy for chest percussion, vibration, and postural drainage, D. Pulseless Ventricular tachycardia
Nurse Dan should concentrate on the following EXCEPT: 38. This is considered as the most important intervention for VF/ pulseless VT with
A. Amount of food and fluid consumed during the last meal before treatment the greatest effect on survival to hospital discharge?
B. Respiratory rate, breath sounds, and location of congestion A. Epinephrine
C. Instructing the client’s relatives on how to perform the procedure B. Defibrillation
D. Doctor’s orders regarding position restrictions and the client’s ability to tolerate C. Oxygen
lying flat – tolerance of positions D. Amiodarone
34. When preparing Tommy for postural drainage and percussion, which of the 39. Which of the following is the most important step to restore oxygenation and
following is a special consideration? ventilation for an unresponsive, breathless near-drowning victim?
A. Respiratory rate of 16 to 20 per minute A. Attempt to drain water from the breathing passages by performing the Heimlich
B. Client’s ability to tolerate sitting and lying positions maneuver
C. Absence of signs of infection B. Begin chest compressions
D. Time of the client’s last food and fluid intake C. Provide cervical spine stabilization because a diving accident may have
35. The purpose of chest percussion and vibration is to loosen lung secretions. The occurred
difference between these procedures is: D. Open the airway and begin rescue breathing as soon as possible, even in the
A. Percussion uses only one hand, while vibration uses both hands water
B. Percussion delivers cushioned blows to the chest with cupped palms, while 40. In initiating CPR, nurse Angel knows it is important to do chest compressions
vibration gently shakes secretions loose during the exhalation cycle correctly. What is the correct way of performing chest compressions?
C. In both percussion and vibration, the hands are placed on top of each other A. Compress more than 120 per minute
and move in sync with the client’s breathing rhythm B. The rescuer’s arms should be parallel to the victim’s body
D. Percussion slaps the chest to loosen secretions, while vibration shakes the C. Compress for 5 cycles at 30 compressions each cycle
secretions during inhalation D. Depth of compressions should be 4 inches
Situation: Nurse Angel is doing her rounds in the ward, when someone called a code. HIGH QUALITY CPR INCLUDES:
She rushed immediately to the bedside. ✓ 100-120 compressions per minute at 5-6 cm (2-2.4 inches)
36. Nurse Angel grabs the defibrillator and rush to the room where resuscitation is in ✓ Keep hand in contact to chest
progress. A colleague has started CPR and confirms the patient is in ventricular ✓ Allow full chest recoil
tachycardia. As she begins to attach the AED, she notices a transdermal medication ✓ 1 cycle of 30 compressions, then provide 2 full one-second > breaths. Check for
patch on the victim's upper right chest, exactly where she needs to place an AED chest rise in between breaths
electrode pad. What is the most appropriate action? ✓ Rescuer arms should be over their hands, arms perpendicular to victim's body, and
A. Ignore medication patch and place the electrode pad in the standard position elbows locked.
B. Avoid medication patch and place second electrode pad on victim’s back Situation: Proper positioning is important to conduct nursing activities.
41. The best position for any procedure that involves vaginal and cervical examination C. Offering a weekly ZUMBA class to young adults with a strong family history of
is high cholesterol – should not be diagosed
A. Dorsal recumbent D. Vaccinating children with anti-rabies after being bitten by a stray do
B. Side lying 47. The Cardinal Santos Medical Center often holds free health programs in the
C. Supine outpatient department. This month of May, they are holding free Pap smears to the
D. Lithotomy – legs on stirrups first 10 registrants each day. This is an example of what kind of prevention?
42. Which of the following is the best position for the client to assume if the back is to A. Primary
be examined by the nurse? B. Secondary – diagnostic, screening, treatment
A. Standing C. Tertiary
B. Supine D. Quaternary
C. Side lying 48. Prior to discharge, nurse Joey discusses with the Santos family the diet
D. Prone modifications their grandmother has to make to prevent further disease progression of
43. In palpating the client’s abdomen, which of the following is the best position for her ESRD. This is a sample of what kind of prevention?
the client to assume? A. Primary
A. Dorsal recumbent – relaxes abdomen B. Secondary
B. Side lying C. Tertiary – diagnosed, activities increase quality of life, rehab, pain
C. Supine management, preventing complications from treatment
D. Lithotomy D. Quaternary
44. Rectal examination is done with a client in what position? 49. Which of the following is an example of tertiary prevention?
A. Dorsal recumbent A. Implementing smoking ordinances in the city to prevent risk of lung cancer
B. Sim’s position – initial = left sim’s position = to full soaked descending colon; B. Promoting use of seat belts to prevent car accidents
full enema = left sim’s position to supine to right sided sim’s position C. Screening of hearing and vision loss in the community center among old adults
C. Supine D. Regular pain management for patient undergoing chemotherapy – increase
D. Lithotomy quality of life
45. One of the patients diagnosed with COPD is having difficulty breathing. Which is 50. A patient experienced a myocardial infarction four weeks ago and is currently
the best position to achieve maximal lung expansion? participating in the daily cardiac rehabilitation sessions at the local fitness center. In
A. High fowlers what level of prevention is the patient participating?
B. Semi fowlers A. Primary
C. Supine B. Secondary
D. Orthopneic – tripod position C. Tertiary – increase quality of life
Situation: Understanding the factors that influence the healthcare delivery system D. Quaternary
enables nurses to adapt to changes, devise improved methods of providing care, and Situation: Proper communication and responses is essential in every nurse-patient
develop new nursing roles. relationship.
46. Cost-effective healthcare prioritizes the primary prevention of illness. Which of the 51. The nurse is caring for a client who is 4 days post-op for a transverse colostomy.
following is an example of a primary prevention activity? “health promotion and disease The client is ready for discharge and asks the nurse to empty his colostomy pouch.
prevention” What is the best response by the nurse?
A. Administering first aid treatment for a patient bitten by a snake A. You should be emptying the pouch yourself.
B. Providing home modifications to help a client adapt their home environment B. Let me demonstrate to you how to empty the pouch.
following a stroke C. What have you learned about emptying your pouch?
D. Show me what you have learned about emptying your pouch 57. A barangay had an outbreak of German measles. To prevent congenital rubella,
52. A 3-year-old child has tympanostomy tubes in place. The child’s parent asks the what is the BEST advice that you can give to women in the first trimester of pregnancy
nurse if he can swim in the family pool. The best response from the nurse is: in the barangay? Rubella vaccine = passive active = inactivated attenuated virus
A. Your child should not swim at all while the tubes are in place. A. Advise them on the signs of German measles.
B. Your child may swim in your own pool but not in a lake or ocean. B. Avoid crowded places, such as markets and movie houses.
C. Your child may swim if he wears ear plugs. C. Consult at the health center where rubella vaccine may be given.
D. Your child may swim anywhere. D. Consult a physician who may give them rubella immunoglobulin – passive
53. The feeling of trust can best be established by the nurse during the process of the artificial
development of a nurse-client relationship by which of these characteristics? 58. You were invited to be the resource person in a training class for food handlers.
A. Reliability and kindness Which of the following would you emphasize regarding prevention of staphylococcal
B. Demeanor and sincerity food poisoning?
C. Honesty and consistency – respect, faith, caring A. All cooking and eating utensils must be thoroughly washed.
D. Sympathy and appreciativeness B. Food must be cooked properly to destroy staphylococcal microorganisms.
54. A client, recovering from alcoholism, asks the nurse, “What can I do when I start C. Food handlers and food servers must have a negative stool examination
recognizing relapse triggers within myself?” How might the nurse best respond? result.
A. When you have the impulse to stop in a bar, contact a sober friend and talk D. Proper handwashing during food preparation is the best way of preventing the
with him. condition.
B. Go to an AA meeting when you feel the urge to drink. 59. In a mothers’ class, you discussed childhood diseases such as chicken pox. Which
C. It is important to exercise daily and get involved in activities that will cause you of the following statements about chicken pox is correct?
not to think about drug use. A. The older one gets, the more susceptible he becomes to the complications of
D. Let’s talk about possible options you have when you recognize relapse triggers chicken pox.
in yourself – self-evaluation and problem solving B. A single attack of chicken pox will prevent future episodes, including conditions
55. One hour before the first treatment is scheduled, the client becomes anxious and such as shingles.
states he does not wish to go through with electroconvulsive therapy. Which response C. To prevent an outbreak in the community, quarantine may be imposed by
by the nurse is most appropriate? health authorities.
A. I’ll go with you and will be there with you during the treatment. D. Chicken pox vaccine is best given when there is an impending outbreak in the
B. You’ll be asleep and won’t remember anything. community.
C. You have the right to change your mind. You seem anxious. Can we talk about 60. Complications to infectious parotitis (mumps) may be serious in which type of
it? clients?
D. I’ll call the health care provider to notify them of their decision. A. Pregnant women
Situation: Infection control measures are done specifically in different types of B. Elderly clients
conditions including proper management to treat diseases, and preventive actions. C. Young adult males – epididymitis + orchitis = sterility; female: pelvic
56. Antiretroviral agents, such as AZT, are used in the management of AIDS. Which inflammatory disease
of the following is NOT an action expected of these drugs. D. Young infants
A. They prolong the life of the client with AIDS. 61. The youngest child of the Yen family is vomiting every after feeding. She is then
B. They reduce the risk of opportunistic infections diagnosed with pyloric stenosis. This is classified as a:
C. They shorten the period of communicability of the disease. A. Health threat – risk factors
D. They are able to bring about a cure of the disease condition. B. Health deficit – (+) disease
C. Foreseeable crisis – anticipatory problem; milestone
D. Stress point – biglaan 67. The purpose of this law is to mandate compulsory basic immunization for infants
62. Mr. and Mrs. Tanseco are discussing their financial budget for their son’s college and children below eight years of age in the Philippines. The law aims to promote the
fees. They are also planning to give him enough allowance for him to afford healthy welfare of children and provide them with opportunities for a useful and happy life.
meals, however this would entail the to cut back on some of their current expenses. A. PD 651
They are experiencing what kind of problems? B. PD 825
A. Health threat C. PD 856
B. Health deficit D. PD 996
C. Foreseeable crisis 68. This law is a national policy that mandates the Philippine government to
D. Stress point comprehensively address the needs of Filipino citizens when it comes to responsible
63. Kiara recently gave birth and wants to commit to be a full-breastfeeding mother. parenthood and reproductive health.
However, she does not know what to do. You plan to teach her the key points of A. RA 10028
breastfeeding. B. RA 10121
A. Wellness potential C. RA 10152
B. Health deficit D. RA 10354
C. Foreseeable crisis 69. The State commits itself to promoting the well-being of people by ensuring that:
D. Stress point mental health is valued, promoted and protected; mental health conditions are treated
64. In a recent house inspection, nurse Mario saw few cockroaches and rodents and prevented; timely, affordable, high-quality, and culturally- appropriate mental
running around in the kitchen. He is especially concerned as the rats were near the health care is made available to the public. This law is the?
toddler’s snacks and formula milk. He raises this concern to the parents. A. RA 10653
A. Health threat. B. RA 10871
B. Health deficit C. RA 10912
C. Foreseeable crisis D. RA 11036.
D. Stress point 70. This act provides for the creation of a national program of air pollution
65. The COVID-19 pandemic led to the loss of numerous job opportunities and management focusing primarily on pollution prevention; for the promotion of mass
positions across different fields of specializations. Mang Karlos lost his primary source media communication in order to create social awareness and active participation in
of income as a taho vendor since people are not allowed to leave their residences. air quality planning and monitoring.
What kind of problem is this? A. RA 8504
A. Health threat B. RA 8749
B. Health deficit C. RA 8976
C. Foreseeable crisis D. RA 9211
D. Stress point. RA 9262 Violence against Women and Children
66. This Law acknowledges that women who have retaliated against their partner or RA 9288 Newborn Screening Act of 2004
who commit violence as a form of self-defense may have suffered from Battered RA 9482 Anti - Rabies Act of 2007
Woman Syndrome (BWS). Any victim who suffers from BWS should be diagnosed by RA 7719 Blood Services Act of 1994
a Psychiatric expert or a clinical psychologist. PD 651 Period of registering births
A. RA 9262. requires penalty for improper disposal of garbage and other forms of
PD 825
B. RA 9288 uncleanliness
C. RA 9482 PD 856 CODE OF SANITATION
D. RA 7719
Compulsory Immunization of all children below 8 years of age against B. Justice.
PD 996
the six childhood immunizable diseases C. Autonomy
RA 10028 Expanded Breastfeeding Act of 2009 D. Veracity
RA 10121 Philippine Disaster Risk Reduction and Management Act (NDRRMC) 74. When giving a high alert medication, nurse Paolo, despite his many years of
MANDATORY INFANT AND CHILDREN HEALTH IMMUNIZATION experience, double checks the order and medication with another nurse. He exhibits
RA 10152
ACT OF 2011 which of the following?
RA 10354 Responsible Parenthood and Reproductive Health Act Of 2002 A. Fidelity
RA 10653 Graphic Health Warnings Law B. Justice
RA 10871 Basic Life Support Training Act C. Beneficence
RA 10912 Continuing Professional Development Act of 2016 D. Nonmaleficence
RA 11036 Mental Health Act
75. Nurse Enzo is prepping his patient before they go into surgery. Changes were
RA 8504 Philippine AIDS Prevention and Control
made and the patient is to be sedated during the procedure. His patient suddenly
RA 8749 CLEAN AIR ACT OF 2000
exclaims, “I am scared as to how the anesthesia will affect me. Will I be asleep the
RA 8976 Philippine Food Fortification Act
whole time? How long will I be out?” He then calls for the anesthesiologist to have
RA 9211 TOBACCO REGULATION ACT
them explain the sedation plans of the procedure, and to secure the consent. His
Situation: Each member of the profession adheres to a set of core ethical principles
patient expresses their relief, and agrees to the plan. What does he exhibit?
in nursing. The nursing code of ethics helps nurses make good judgment calls, provide
A. Fidelity
high-quality care, and support patients and their loved ones emotionally, physically,
B. Justice
and mentally.
C. Beneficence.
71. Nurse Bea diligently performs her job. She attends to her patients’ needs, keeps
D. Nonmaleficence
them warm and comfortable, and maintains their safety at all times. Her actions help
Situation: Programs of the DOH are implemented across the Philippines to help the
her to maintain the trust of her patients to her. She exhibits which of the following?
Filipino achieve Universal health care
A. Fidelity – keep your word
B. Justice – fairness 76. These are the key strategies of MNCHN, except:
A. Ensuring universal access to and utilization of MNCHN core package services
C. Beneficence – doing good at all times (client advocates) = benefits the patient
and interventions directed not only to individual women of reproductive age
D. Veracity – open and honest
and newborns at different stages of the life cycle.
72. The patient makes a decision that is against the medical advice of her physician.
All advantages and disadvantages were mentioned, and still, she chooses to go home B. Establishment of a service delivery network at only the primary level of care. –
all level of care
against the medical advice. She is no inebriated or under influence of any substance.
C. Organized use of instruments for health systems development
You advocated her decision, but still provided essential discharge instructions. This
D. Rapid build-up of institutional capacities of DOH and PhilHealth.
exhibits?
77. Nurse Maloi advocates her clients to have their prenatal visits. What is the
A. Fidelity
recommended minimum number of visits throughout the course of pregnancy?
B. Justice
A. 6
C. Autonomy
B. 8 – WHO
D. Veracity
C. 4 – DOH
73. Typhoon Karina hits the cities with strong winds, and rains – destructing the cities
D. 5
of Japan. Nurse Kate ensures that each families get relief packs during the rescue
78. Essential newborn care is also practiced in every delivery to ensure that all needs
mission. She also prioritizes the people who are in need of immediate attention than
of the newborn are met, and protection is given. These are the interventions done
those who are still able to walk and do their activities of daily living.
within the first 90 minutes, except:
A. Fidelity
A. Immediate thorough drying B. Portal of entry
B. Skin-to-skin contact between mother and newborn. C. Susceptible host – boost immune system
C. Early Cord clamping 1 minute after birth is recommended – delayed cord D. Reservoir
clamping 1-3 minutes after birth 83. You always change your gloves and practice hand hygiene before and after each
D. Early initiation of breastfeeding (within 1 hour after birth) patient. You minimize the risk in what part of the chain?
✓ Rooming in (non-separation of baby from mother) A. Mode of transmission
79. The Family Planning program started in the 1970s as a family planning service B. Portal of exit
delivery component to achieve fertility reductions. FP is means to prevent high- risk C. Susceptible host
pregnancies brought about by the following conditions, except: D. Reservoir
A. Being too young (less than 20 years old) or too old (over 33 years old) – should 84. You make sure to collect all instruments every after surgery to have them sterilized.
be 34 y/o You know that this minimizes risk in?
B. Having had too many (4 or more) pregnancies A. Pathogen / Causative agent
C. Having closely spaced (too close) pregnancies (less than 36 months) B. Portal of entry
D. Being too ill or unhealthy/ too sick or having an existing disease or disorder like C. Susceptible host
iron deficiency anemia. D. Reservoir
80. Nurse Maloi also advocates for family planning fit according to the lifestyle and 85. You practice sterility in placing sterile dressings on surgical wounds. You know that
preference of the clients. She advocates the following barrier family planning methods, this minimizes risk in?
except: A. Pathogen / Causative agent
A. Condom – males B. Portal of entry
B. Intrauterine device (-); sterile inflammation C. Susceptible host
C. Cervical cap – females D. Reservoir
D. Diaphragms – females Situation: Malaria is a life-threatening parasitic disease transmitted to humans
Situation: The spread of an infection within a community is described as a “chain,” through the bite of infected mosquitoes. The Department of Health on February 7,
several interconnected steps that describe how a pathogen moves about. Infection 2023 reported that all provinces in the country have achieved malaria-free status
control and contact tracing are meant to break the chain, preventing a pathogen from except for Palawan.
spreading. 86. This agent causes the most serious type of malaria because of high parasitic
81. This is any person, animal, arthropod, plant, soil, or substance (or combination of densities in blood, and causes malignant tertian malaria. This is most common in the
these) in which an causative agent normally lives and multiplies, on which it depends Philippines.
primarily for survival, and where it reproduces in such numbers that it can be A. Plasmodium falciparum
transmitted to a susceptible host. B. Plasmodium vivax
A. Pathogen / Causative agent – cause of disease C. Plasmodium ovale
B. Portal of entry – This is how the infection enters another individual. D. Plasmodium malaria
C. Susceptible host – A person or animal not possessing sufficient resistance to 87. Clinical manifestations of malaria in three stages. This stage happens in 4-6 hours
a particular infectious agent to prevent contracting infection or disease when with recurring high-grade fever, severe headache, vomiting, abdominal pain, and blue
exposed to the agent face.
D. Reservoir – place in which infection lives A. Cold Stage – 10-15 mins = chills (1)
82. By encouraging rest and balance in our lives, you try to minimize risks in what part B. Warm stage
of the chain of infection? C. Hot stage – (2)
A. Pathogen / Causative agent D. Diaphoretic stage – sweating (3)
88. Conducting the early diagnosis and prompt treatment for malaria is done to 90. In Integrated Management of Childhood Illness, severe conditions generally
prevent further spread of the disease. This method is based on the examination of require urgent referral to a hospital. Which of the following severe conditions DOES
blood smear of the patient through a microscope. NOT always require urgent referral to a hospital?
A. Early diagnosis A. Mastoiditis
B. Clinical method B. Severe dehydration – give IV therapy – oresol/NGT – oresol/oral
C. Microscopic method - S/sx + history C. Severe pneumonia
D. Chemoprophylaxis – administration of meds to prevent infection D. Severe febrile disease
89. Sustainable Preventive and Vector Control Measures refer to the adoption of TWO OF THE SEVERE If child has no other severe classification:
FOLLOWING SIGNS: DEHYDRATION ✓ PLAN C: Give fluid for severe dehydration.
measures for the prevention and control against the malaria parasite and the mosquito
✓ Lethargic or If child also has another severe classification:
vector. This is the cutting of the vegetation overhanging along stream banks to expose unconscious ✓ Refer URGENTLY to hospital with mother
the breeding stream to sunlight, rendering it unsuitable for mosquito vector habituation. ✓ Sunken eyes giving frequent sips of ORS on the way.
A. Insecticide-Treatment of Mosquito Net ✓ Not able to drink or ✓ Advise the mother to continue
drinking poorly breastfeeding.
B. House Spraying ✓ Skin pinch goes If child is 2 years older and there is cholera in
C. On stream seeding back very slowly your area, give antibiotic for cholera
D. On stream clearing. (DOXYCYCLINE).
✓ Insecticide - Treatment of Mosquito Net 91. A client was diagnosed as having Dengue fever. You will say that there is slow
o This involves the soaking of the mosquito net in an insecticide solution and capillary refill when the color of the nailbed that you pressed does not return within
allowed to dry. Such treated net is used as a protective measure against the how many seconds?
vector mosquito during sleeping time at night. Insecticide- treated curtains may A. 3
be used in areas where they are more culturally acceptable than mosquito nets. B. 5
✓ House Spraying C. 8
o This is the application of insecticide on the indoor surfaces of the house D. 10
through spraying. 92. A 3-year-old child was brought by his mother to the health center because of fever
✓ On Stream Seeding of 4-day duration. The child had a positive tourniquet test result. In the absence of
o This involves the construction of bio-ponds for fish propagation which shall be other signs, which is the most appropriate measure that the PHN may carry out to
the responsibility of the LGUs and their corresponding communities. The prevent Dengue shock syndrome?
numbers of bio-ponds to be constructed as sources of larvivorous fish, for each A. Insert an NGT and give fluids per NGT.
malaria-endemic municipality, will depend on the number of streams to be B. Instruct the mother to give the child Oresol.
seeded with the propagated larvivorous fish. To be effective, about 2-4 fish per C. Start the patient on intravenous fluids STAT.
sq.m is needed for an immediate impact and about 200-400 fish per ha. is D. Refer the client to the physician for appropriate management.
needed for a delayed effect. DANGER SIGNS
✓ On Stream Clearing V – vomiting
o This is the cutting of the vegetation overhanging along stream banks to expose U – Unable to drink/feed
the breeding stream to sunlight, rendering it unsuitable for mosquito vector L – lethargy
habituation. C – convulsions
Situation: The Integrated Management of Childhood Illness (IMCI) is an integrated 93. A 5-month-old infant was brought by his mother to the health center because of
approach to child health that focuses on the well-being of the whole child. IMCI aims diarrhea occurring 4 to 5 times a day. His skin goes back slowly after a skin pinch and
to reduce death, illness and disability, and to promote improved growth and his eyes are sunken. Using the IMCI guidelines, you will classify this infant in which
development among infants and children aged less than 5 years. category?
A. No signs of dehydration INTERVENTION FOR INFUSION REACTION
B. Some dehydration. SLOW DOWN STOP INFUSION
C. Severe dehydration Circulatory overload 1. Anaphylactic/Allergic
D. The data is insufficient. ✓ Cool extremities ✓ DOB, Hives
94. Based on assessment, you classified a 3-month-old infant with the chief complaint ✓ Crackles in lungs 2. Febrile – fever
of diarrhea in the category of SOME DEHYDRATION. Based on IMCI management 3. Hemolytic
guidelines, which of the following will you do? 75 ml/kg = appropriate / age ✓ Blood transfusion
✓ Flank pain
A. Bring the infant to the nearest facility where IV fluids can be given.
4. Air embolism
B. Supervise the mother in giving 200 to 400 ml. of Oresol in 4 hours.
✓ Chest petechiae
C. Give the infant’s mother instructions on home management.
98. She will now proceed to give the appropriate feeding for her patient. Which of the
D. Keep the infant in your health center for close observation.
TWO OF THE SOME ✓ PLAN B: Give fluid, zinc supplements and food following is inappropriate nursing action when administering NGT feeding?
FOLLOWING SIGNS: DEHYDRATION for some dehydration. A. Place the feeding 20 inches above the point of insertion of NGT – 12 -18 inches
✓ Restless and ✓ Advise mother when to return immediately only
irritable ✓ Follow-up in S days if not improving
✓ Sunken eyes If child has severe classification:
B. Introduce the feeding slowly
✓ Drinks eagerly; ✓ Refer URGENTLY to hospital with other C. Instill 60ml of water into the NGT after feeding
thirsty giving frequent sips of ORS on the way. D. Assist the patient in fowler's position
✓ Skin pinches goes ✓ Advise the mother to continue
back slowly breastfeeding.
99. During application of medication into the ear, which of the following is inappropriate
95. A mother is using Oresol in the management of diarrhea of her 3-year-old child. nursing action?
She asked you what to do if her child vomits. You will tell her to: A. In an adult, pull the pinna upward
A. Bring the child to the nearest hospital for further assessment. B. Instill the medication directly into the tympanic membrane. – may puncture the
B. Bring the child to the health center for intravenous fluid therapy. eardrum
C. Bring the child to the health center for assessment by the physician. C. Warm the medication at room or body temperature
D. Let the child rest for 10 minutes then continue giving Oresol more slowly. D. Press the tragus of the ear a few times to assist flow of medication into the ear
Situation: Nurse Ysabel reviews the fundamentals as she studies for the board exam. canal
96. What is the order of the nursing process? 100. Postural drainage to relieve respiratory congestion should take place:
A. Assessing, diagnosing, implementing, evaluating, planning A. Before meals – risk for aspiration
B. Diagnosing, assessing, planning, implementing, evaluating B. After meals
C. Assessing, diagnosing, planning, implementing, evaluating. C. At the nurse's convenience
D. Planning, evaluating, diagnosing, assessing, implementing D. At the patient's convenience
97. A client is receiving 115 ml/hr of continuous IVF. Nurse Ysabel notices that the COMPREHENSIVE PHASE - RECALLS 1 (NP2)
venipuncture site is red and swollen. Which of the following interventions would she Situation:
perform first? Pregnancy is a dangerous milestone. Nurse Jeara intends to lessen pregnancy
A. Stop the infusion complications.
B. Call the attending physician 1. Nurse Jeara is evaluating a pregnant patient diagnosed with abruptio placentae.
C. Slow that infusion to 20 ml/hr Which symptoms should she anticipate observing? Select all that apply.
D. Place a cold towel on the site I. Uterine irritability
II. Uterine tenderness
III. Painless vaginal bleeding – painful
IV. Abdominal and low back pain
V. Strong and frequent contractions 6. She is caring for a client in labor. She determines that the client is beginning in the
VI. Nonreassuring fetal heart rate patterns – uteroplacental insufficiency 2nd stage of labor (ends in birth) when which of the following assessments is noted?
A. I, II, III, IV A. The client begins to expel clear vaginal fluid
B. I, II, IV, V B. The contractions are regular
C. I, II, IV, VI C. The membranes have ruptured
D. II, III, IV D. The cervix is dilated completely.
2. While caring for a client in the active stage of labor, she observes a late deceleration 7. Nurse Jessa is caring for a client in labor and is monitoring the fetal heart rate
on the fetal monitor strip. What immediate action should she plan? patterns. The nurse notes the presence of episodic accelerations on the electronic
A. Document the findings. fetal monitor tracing. Which of the following actions is most appropriate?
B. Prepare for immediate birth. A. Document the findings and tell the mother that the monitor indicates fetal well-
C. Increase the rate of oxytocin infusion. being.
D. Administer oxygen to the client via face mask – uteroplacental insufficiency; B. Take the mothers vital signs and tell the mother that bed rest is required to
decrease blood flow conserve oxygen.
3. She is preparing to assess a client in her second trimester of pregnancy. When C. Notify the physician or nurse mid-wife of the findings.
measuring fundal height, what should she expect regarding its relationship to D. Reposition the mother and check the monitor for changes in the fetal tracing
gestational age? 8. She is caring for a client with abruptio placenta and is monitoring the client for
A. It is less than gestational age. disseminated intravascular coagulopathy. Which assessment finding is least likely to
B. It corresponds with gestational age – 3rd tri – not corresponds = fetus bababa be associated with disseminated intravascular coagulation?
C. It is greater than gestational age. A. Swelling of the calf in one leg – not a sign for DIC but Thrombophlebitis
D. It has no correlation with gestational age. B. Prolonged clotting times
4. A pregnant client informs her that she noticed wetness on her peripad and C. Decreased platelet count
discovered some clear fluid. Upon inspecting the perineum, Jeara observes the D. Petechiae, oozing from injection sites, and hematuria
umbilical cord. What should the nurse do immediately? 9. A client is admitted to the birthing suite in early active labor. The priority nursing
A. Monitor the fetal heart rate. intervention on admission of this client would be:
B. Notify the primary health care provider. A. Auscultating the fetal heart – assess fetal well-being
C. Transfer the client to the delivery room. B. Taking an obstetric history
D. Place the client in the Trendelenburg position – relieve cord compression C. Asking the client when she last ate
5. She is giving advice to a patient in the third trimester of pregnancy regarding D. Ascertaining whether the membranes were ruptured
heartburn management techniques. Which instructions should she give to the patient? 10. A postpartum patient was in labor for 30 hours and had ruptured membranes for
A. Sip some hot tea or milk. 24 hours. For which of the following would nurse Jessa be alert?
B. Make use of sodium-containing antacids. A. Endometritis – inflammation of the uterine endometrium
C. Consume fatty meals only once a day in the morning. B. Endometriosis – a disease in which tissue similar to the lining of the uterus
D. Rather than eating short, regular meals, eat three big meals a day. grows outside the uterus
HEARTBURN MANAGEMENT TECHNIQUES C. Salpingitis – inflammation of the fallopian tubes, caused by bacterial infection
1. Avoid fatty, fried, coffee, stimulants D. Pelvic thrombophlebitis – a blood clot that blocks one or more pelvic veins
2. EAT small frequent feeding Situation: Nurse Tommy, a nurse in Barangay Marilag Health Center, will conduct
Situation: Nurses Jessa is assigned in high-risk maternity ward. Here are some cases Health education to the residents of the Barangay on Reproductive Health.
which she handles for the morning shift.
11. A prenatal client is suspected of having iron deficiency anemia. During the 14. A client with a family history of heart disease presents to the primary health care
assessment, which finding should the nurse he expects to observe as a result of this provider's office with a request to start oral contraceptive therapy for birth control. What
condition? important topic should Nurse Tommy ask the client about next?
A. Dehydration A. Smoking – potent vasoconstrictor = (+) clotting
B. Overhydration B. Regular exercise
C. A high hematocrit level C. Low cholesterol diet
D. A low hemoglobin level D. Alternative methods of contraception
12. A prenatal client has been diagnosed with a vaginal infection caused by Candida 15. Nurse Tommy teaches a pregnant client to perform Kegel exercises. Which
albicans. What should he expect to observe during the assessment? statement by the client indicates an understanding of the purpose of these types of
A. Costovertebral angle pain exercises?
B. No observable signs A. “The exercises will help reduce backache.” – pelvic tilt
C. Pain, itching, and vaginal discharge – white creamy B. “The exercises will help prevent ankle edema.” – leg elevations
D. Proteinuria, hematuria, and hypertension – Upper UTI C. “The exercises will help strengthen the pelvic floor.”
13. For a client starting oral medication, he gives instructions regarding the use of oral D. “The exercises will help prevent urinary tract infections.” – drinking 6-8 oz of
contraceptives. Which of the client's statements suggests that more instruction is fluids
necessary? Situation: Nurse Dan is assigned at the postpartum ward. The following questions
A. "Every day at the same time, I will take one pill." (+) refer to postpartum patients.
B. "I have to take the missed pill as soon as I remember it if I miss it." (+) 16. Nurse Dan is caring for a postpartum client. He should suspect endometritis if
C. "After I begin using these, I won't need to use any other birth control methods.” which of the following is observed?
D. “I will take both pills as soon as I remember if I had missed two pills, and then A. Breast engorgement
two tablets the following day.” (+) B. Elevated white blood cell count.
1. If the pill omitted was one of the placebo ones, ignore it and just take the next pill C. Lochia rubra on the second day postpartum
on time the next day. D. Fever over 38° C (100.4° F) starting 2 days postpartum
2. If you forgot to take one of the active pills, take it as soon as you remember. 17. He gives instructions to a new mother who is preparing to breastfeed her newborn.
Continue the following day with your usual pill schedule. Doing so might mean While observing the first breastfeeding session, he determines that further teaching is
taking two pills on one day if you don't remember until the second day, but that's needed if the new mother performs which technique?
all right. Missing one pill this way should not initiate ovulation. A. Turns the newborn onto his side, facing the mother
3. If you miss two consecutive active pills, take two pills as soon as you B. Tilts up the nipple or squeezes the areola, pushing it into the newborn's mouth
remember. Then, continue the following day with your usual schedule. You may – (-) latch, flow of milk
experience some breakthrough bleeding (vaginal spotting) with two forgotten pills. C. Draws the newborn further onto the breast when the newborn opens his mouth
Do not mistake this bleeding for your menstrual flow. Missing two pills may allow D. Inserts a clean finger into the side of the newborn's mouth to break the suction
ovulation to occur, so an added contraceptive such as a spermicide should be used before removing the newborn from the breast
for the remainder of the month. 18. A breastfeeding client, ten days postpartum, calls the postpartum unit, reporting a
4. If you miss three or more pills in a row, throw out the rest of the pack and painful, reddened breast and a fever. Considering the client's symptoms, what
start a new pack of pills. You might not have a period because of this routine and guidance should he provide?
should use extra protection until 7 days after starting a new pack of pills A. "Continue breastfeeding from the unaffected breast only."
5. If you think you might be pregnant, stop taking pills and notify your healthcare B. "Cease breastfeeding as it's likely you have an infection."
provider. C. "Contact your healthcare provider as medication may be necessary."
D. "Continue with breastfeeding as this reaction is typical in breastfeeding B. Uterine tone
mothers." C. Blood pressure – will increase
19. When a breastfeeding mother mentions experiencing nipple soreness, nurse Dan D. Deep tendon reflexes
offers guidance on alleviating the discomfort. Which statement from the mother 24. A registered nurse is a preceptor for a new nurse and is observing the new nurse
demonstrates comprehension of the instructions? organize the client assignments and prioritize daily tasks. The registered nurse should
A. “I should refrain from changing breastfeeding positions to toughen the nipple.” intervene if the new nurse implements which action?
B. “I should temporarily halt nursing to give the nipples time to heal.” A. Provides times for staff meals
C. “I should breastfeed less often and replace some feedings with bottle feeding B. Gathers the supplies needed for a task
until the nipples feel better.” C. Combines all tasks for clients in one list
D. “I should position my baby so her ear, shoulder, and hip are aligned and her D. Documents task completions at the end of the day – done continuously
stomach is against me.” throughout the day
20. A new breastfeeding mother, suffering from breast engorgement, receives 25. When the nurse manager encourages staff to provide input in the decision
guidance on managing the condition. Which statement from the mother indicates to making-process, which leadership style is being demonstrated?
nurse Dan that she comprehends the strategies for alleviating discomfort from breast A. Autocratic – task oriented: leader = will make decisions
engorgement? B. Situational – depends on situations
A. "I will breastfeed using just one breast." C. Democratic
B. "I will use cold compresses on my breasts." – warm compress/warm showers D. Laissez-faire – leader == facilitator
C. "I will refrain from wearing a bra while my breasts are engorged." – wear Situation: Nurse Lyka, a maternity nurse, firmly believes that everyone should receive
supporting bra good RH education.
D. "I will massage my breasts before feeding to stimulate milk letdown." 26. She is educating a pregnant client about the physiological impacts and hormonal
Situation: It is essential to perform prompt nursing process to all patients. Nurse shifts during pregnancy. What information should she convey regarding the function of
Hannah devotes herself in performing her best in the maternity ward. estrogen?
21. During an assessment of a mother who recently gave birth to a healthy newborn, A. Estrogen maintains the uterine lining to facilitate implantation. – Progesterone
where should the nurse anticipate finding the fundus positioned? B. Estrogen boosts glucose metabolism and converts glucose into fat. – Human
A. On the right side of the abdomen – full bladder placental lactogen
B. Level with the umbilicus C. Estrogen inhibits the regression of the corpus luteum, maintaining
C. Above the umbilicus – (+) blood clots progesterone production until the placenta forms. – Human Chorionic
D. One finger's breadth above the symphysis pubis – involution Gonadotrophin
22. A new mother received an intramuscular injection of methylergonovine maleate D. Estrogen stimulates uterine growth to create a conducive environment for the
following delivery. The nurse comprehends that this medication was administered for fetus and primes the breasts for lactation
what purpose? 27. She is preparing to conduct a class on testicular self-examination (TSE) at a local
A. To decrease uterine contractions. high school, should incorporate which guidance for the attendees?
B. To prevent postpartum hemorrhage – increase contraction A. Conduct the self-examination every two months. – Every month
C. To sustain normal blood pressure. B. Perform the self-examination following a cold shower. – During/after shower
D. To diminish the amount of lochia drainage. C. Anticipate slight discomfort during the self-examination. – Not be painful
23. Methylergonovine maleate is prescribed for a woman who has just delivered a D. Gently roll the testicle between the thumb and forefinger.
healthy newborn. Which is the priority assessment to complete before administering 28. She is teaching a group of females how to prevent pelvic inflammatory disease
the medication? (PID). What instruction should the nurse include?
A. Lochia A. To douche monthly
B. To avoid unprotected intercourse – main cause of PID: STI; thus avoiding A. Hypoglycemia
transmission of virus B. Respiratory distress syndrome – pre-term
C. To use only ultra-low dose oral contraceptive pills C. Meconium aspiration syndrome – (+) swallowed by fetus
D. To consult with a gynecologist regarding the placement of an intrauterine D. Transient tachypnea of the newborn – commonly found in NB via CS
device (IUD) 34. The nurse is evaluating a 3-day-old preterm neonate diagnosed with respiratory
29. She is providing discharge teaching to a client after a vasectomy. Which statement distress syndrome (RDS). Which assessment finding suggests improvement in the
by the client indicates the need for further teaching neonate's respiratory condition?
A. “I can use a scrotal support if I need to.” A. Edema of the hands and feet
B. “I don’t need to practice birth control any longer.” – they have to reach B. Urine output of 3 mL/kg/hour
azoospermia first (no sperm cells) C. Presence of a systolic murmur – Patent ductus arteriosus
C. “I can resume sexual intercourse whenever I want.” D. Respiratory rate ranging from 60 to 70 breaths per minute – >60 bpm =
D. “I can use an ice bag and take an analgesic for pain or swelling.” tachypnea
30. She asks a student to identify risk factors for and methods of preventing prostate 35. A newborn baby whose mother is Rh negative is told to be admitted to the nursery
cancer. Which statement by the student indicates the need for further teaching? by the nurse. Which move should the Angel make in preparation for the infant's arrival?
A. “Smoking increases the risk for this type of cancer.” A. Determine the blood type of the newborn and direct Coombs results – direct
B. “A high-fat diet will assist in preventing this type of cancer.” – increase the risk Coombs = umbilical cord blood; indirect = mother’s blood
C. “A history of a sexually transmitted infection is a risk for this disease.” B. Acquire from the blood bank the supplies required for an exchange transfusion.
D. “Men more than 50 years old should be monitored with a yearly digital rectal C. Request the installation of a phototherapy unit by calling the maintenance
exam.” department to be taken to the nursery.
Situation: Preterm newborns are considered high risk because of their immaturity to D. To avoid isoimmunization, get a vial of vitamin K from the pharmacy and be
adapt to the extrauterine life. ready to give an injection.
31. On assessment of a newborn being admitted to the nursery, Nurse Angel palpates Situation: Nurse Adolf loves taking care of pediatric patients. Pediatric nursing care
the anterior fontanel and notes that it feels soft. She determines that this finding demands a thorough assessment so that good management may be provided.
indicates which condition? 36. A newborn baby has been diagnosed with imperforate anus. How should Nurse
A. Dehydration – fontanel is sunken Adolf explain this condition to the parents?
B. A normal finding A. The occurrence of fecal incontinence – encopresis
C. Increased intracranial pressure – bulging fontanels B. Inadequate development of the anus.
D. Decreased intracranial pressure C. Difficulty passing infrequent, dry stools – constipation
✓ Anterior = diamond; closes @ 12-18 mos. D. Folding of a portion of the intestine into the lower bowel – intussusception
✓ Posterior = triangle: closes @ 2-3 mos. 37. Which nursing assessment findings indicate normal vital signs in a newborn
32. Nurse Angel is assessing a post term infant. Which physical characteristic should infant?
the nurse expect to observe? A. Pulse, 112; respiratory rate, 24
A. Peeling skin – leather-like skin (desquamation) B. Pulse, 124; respiratory rate, 28
B. Smooth soles without creases – pre-term C. Pulse, 144; respiratory rate, 48 – normal pulse: 120-160
C. Lanugo covering the entire body – pre-term D. Pulse, 164; respiratory rate, 55
D. Thick layer of vernix covering the body – pre-term 38. Nurse Adolf teaches nursing students about mumps. Which clinical manifestation
33. A post term infant delivered vaginally is showing signs of tachypnea, grunting, will the specialist identify as the most common complication of this disease?
retractions, and nasal flaring. Angel interprets these findings as indicative of which A. Pain
condition? B. Nuchal rigidity – aseptic meningitis
C. Impaired hearing A. “Most children do not receive the vaccine until they are 5 years of age.”
D. A red swollen testicle B. “You are still susceptible to rubella, so your toddler should receive the vaccine.”
39. Nurse Adolf is assigned to care for an infant on the first postoperative day after a C. “It is not advised for children of pregnant women to be vaccinated during their
surgical repair of a cleft lip. Which nursing intervention is appropriate when caring for mother’s pregnancy.”
this child’s surgical incision? D. “Your titer supports your immunity to rubella, and it is safe for your toddler to
A. Rinsing the incision with sterile water after feeding receive the vaccine.”.
B. Cleaning the incision only when serous exudate forms 44. The nurse is assigned to care for a hospitalized toddler. Which measure should
C. Rubbing the incision gently with a sterile cotton-tipped swab – damage integrity the nurse plan to implement as the highest priority of care?
of incision A. Providing a consistent caregiver
D. Replacing the Logan bar carefully after cleaning the incision B. Protecting the toddler from injury
40. He is caring for a child diagnosed with Reye’s syndrome. He monitors for C. Adapting the toddler to the hospital routine
manifestations of which condition associated with this syndrome? D. Allowing the toddler to participate in play and diversional activities
A. Protein in the urine 45. A mother of a 3-year-old child asks the nurse what personal and social
B. Symptoms of hyperglycemia – hypoglycemia developmental milestones she can expect to see in her child. The nurse should tell the
C. Increased intracranial pressure mother to expect which findings? Select all that apply.
D. A history of a staphylococcus infection – viral infection I. Begins problem-solving
Reye’s disease = a rare but serious condition that causes swelling in the liver and II. Exhibits sexual curiosity
brain. III. May begin to masturbate
Situation: Growth and development refer to the physical, cognitive, emotional, and IV. Notices gender differences
social changes that occur throughout an individual's lifespan. It encompasses various V. Develops a sense of initiative – 4-5 y/o
aspects of human development, from infancy to adulthood. VI. Develops positive self-esteem through skill acquisition – 6-8 y/o
41. A 10-year-old child is newly diagnosed with type 1 diabetes mellitus. The nurse is A. I, II, III.
planning for home care with the child and the family and determines that which is an B. II, IV
age-appropriate activity for health maintenance? C. II, III, IV
A. Administering insulin drawn up by an adult D. II, III, V
B. Self-administering insulin with adult supervision – (+) cognitive and motor skills Situation: You have recently passed the PNLE and is now hired in a hospital. You
C. Making independent decisions with regard to sliding-scale coverage of insulin started to review the fundamentals
D. Having an adult assist in the self-administration of insulin and glucose 46. What action should you take before performing a venipuncture to start continuous
monitoring intravenous (IV) therapy?
42. A home care nurse is providing instructions to the mother of a toddler regarding A. Apply a cool compress to the area. – vasoconstriction
safety measures in the home to prevent an accidental burn injury. Which statement by B. Inspect the IV solution and check the expiration date.
the mother indicates a need for further instruction? C. Secure a padded arm board above the IV site. – after IV
A. “I need to use the back burners for cooking.” D. Apply a tourniquet below the venipuncture site. – above
B. “I need to remain in the kitchen when I prepare meals.” 47. The doctor prescribes acetaminophen liquid 450 mg orally every 4 hours PRN for
C. “I need to be sure to place my cup of coffee on the counter.” pain. The medication label reads 160 mg/5mL. You prepare how many milliliters (mL)
D. “I need to turn pot handles inward and to the middle of the stove.” to administer one dose?
43. The nurse is reviewing the results of the rubella screening (titer) with a pregnant A. 12 ML
client. The test results are positive, and the client asks if it is safe for her toddler to B. 15ML
receive the vaccine. Which response by the nurse is most appropriate? C. 13ML
D. 14ML
Desired dose/Stock on hand x diluent = ml
48. You are sending an arterial blood gas (ABG) specimen to the laboratory for 52. You are anticipating rapid fluid replacement therapy for the patient. With the
analysis. What information should be included on the laboratory requisition? Select all previously identified percentage of burn injury, how much fluid should be administered
that apply. over the first 24 hours if the patient weighs 72 kg?
I. Ventilator settings A. 9,072 mL
II. List of client allergies B. 7,776 mL
III. Client’s temperature C. 13,968 mL
IV. Date and time the specimen was drawn D. 15,552 mL
V. Any supplemental oxygen the client is receiving PARKLAND FORMULA = 4 milliliters x % TBSA (total body surface area burned) x
VI. Extremity from which the specimen was obtained body weight (kg)
A. I, II, III First 8 hours = 1st half
B. I, II, IV, V Next 16 hours = 2nd half
C. I, III, IV, VI 53. A client who is brought to the emergency department has experienced a burn
D. I, III, IV, V covering greater than 25% of his total body surface area (TBSA). When reviewing the
49. Which arterial blood gas (ABG) values should you anticipate in the client with a laboratory results drawn on the client, which value should you most likely expect to
nasogastric tube attached to continuous suction? note?
A. pH 7.25, Paco2 55, HCO3 24 – resp. acidosis A. Hematocrit 65% (0.65) – increase hemoconcentration due to loss of
B. pH 7.30, Paco2 38, HCO3 20 – met. acidosis intravascular fluid = fluid shifting
C. pH 7.48, Paco2 30, HCO3 23 – resp. alkalosis B. Albumin 4.0 g/dL (40 g/L)
D. pH 7.49, Paco2 38, HCO3 30 – met. alkalosis C. Sodium 140 mEq/L (140 mmol/L)
50. For ensuring client safety, which assessment should you prioritize before D. White blood cell (WBC) counts 6000 mm3 (6 × 109/L)
transitioning a client from liquid to solid food? 54. The client is scheduled for hydrotherapy for a burn dressing change. Which action
A. Bowel sounds should you take to ensure that the client is comfortable during the procedure?
B. Chewing ability A. Ensure that the client is appropriately dressed.
C. Current appetite B. Administer an opioid analgesic 30 to 60 minutes before therapy.
D. Food preferences C. Schedule the therapy at a time when the client generally takes a nap.
Situation: An adult client is admitted to the emergency department after a burn injury. D. Assign unlicensed assistive personnel (UAP) to stay with the client during the
51. The burn initially affected the upper half of the client’s anterior torso, and there procedure.
were circumferential burns to the lower half of both of the arms. The client’s clothes 55. He receives a prescription for a regular diet. Which is the best meal for you to
caught on fire, and the client ran causing subsequent burn injuries to the entire face provide to the client to promote wound healing?
(anterior half of the head), and the upper half of the posterior torso. Using the rule of A. Peanut butter and jelly sandwich, apple, tea
nines, the extent of the burn injury would be what percent? B. Chicken breast, broccoli, strawberries, milk – high protein
A. 27% C. Veal chop, boiled potatoes, Jell-O, orange juice
B. 48.5% D. Pasta with tomato sauce, garlic bread, ginger ale
C. 54% Situation: In every developmental stage, there are different psychosexual stages a
D. 31.5% child must go through and fulfill to meet its needs. As a healthcare professional who
values the holistic development of every individual, nurse Jen provides appropriate
interventions for each life stage.
56. She was asked by a first-time mom how she can provide and meet the needs of 60. The nurse understands that Freud's phallic stage of psychosexual development,
her 3-month-old infant. She tries to fill the crib with toys, but her infant is not responding which compares with Erikson's psychosocial phase of initiative versus guilt, is seen
to it very well. What should she do? best at
A. Place large, colorful teddy bears around the infant’s crib A. Adolescence
B. Provide safe and washable toys such as pacifier – oral stage B. 6 to 12 years
C. Give brightly-colored building blocks to the infant – pre-school C. 3 to 5 1/2 years
D. Give her infant a rattle for her to shake -palmar grasp = 5 months old D. Birth to 1 year
57. Toddlers exhibit signs of readiness for toilet training. Which among the following 61. The nurse understands that problems with dependence versus independence
are not included as its criteria? develop during the stage of growth and development known as:
A. Child is able to stay dry for 2 hours A. Infancy
B. Child is waking up dry from a nap B. School age
C. Child is able to sit, squat, and walk C. Toddlerhood – autonomous = negativism
D. Child is able to wear clothing by themselves – take off clothes by themselves D. Preschool age
SIGNS OF READINESS FOR TOILET TRAINING 62. When planning to teach about the stages of growth and development, what stage
✓ Child is able to stay dry for 2 hours does the nurse indicate as basically concerned with role identifications?
✓ Child is waking up dry from a nap A. Oral stage
✓ Child is able to sit, squat, and walk B. Genital stage
✓ Child is able to remove clothing C. Oedipal stage
✓ Child recognizes urge to defecate or urinate D. Latency stage
✓ Child expresses willingness to please parent 63. The nurse incorporates play in interactions with preschool-aged children,
✓ Child is able to sit on toilet for 5 to 10 minutes without fussing or getting off recognizing that play is essential for their emotional development in the areas of:
58. This type of attachment or complex is present among pre-school children wherein A. Projection
the strong emotional attachment of a preschool boy is present towards the mother, B. Introjection – the unconscious adoption of the ideas or attitudes of others.
and they usually tend to hate the father C. Competition
A. Electra complex – describe a girl's sense of competition with her mother for the D. Independence
affection of her father 64. The nurse understands that the resolution of the Oedipus complex occurs when
B. Oedipus complex – Envy and Jealous are aimed at the father, the object of the child:
the mother's affection and attention A. Rejects the parent of the same sex
C. Persephone complex B. Adopts behaviors of both parents
D. Adolphus complex C. Identifies with the parent of the same sex
59. The nurse observes an 8-year-old child for any signs of psychosexual D. Identifies with the parent of the opposite sex
development. What should they expect from this stage? 65. Surgery can be a very traumatic even for a child. The nurse, when performing
A. a time in which children’s libido appears to be diverted into concrete thinking – preoperative preparation, knows that according to Piaget's stages of cognitive
latency period development, children will experience the greatest fear during the:
B. at the age where they want to fulfill their intimate desires A. Sensorimotor stage
C. at the age where they are ready to learn how to defecate by themselves B. Preoperational stage – difficulty establishing reality and fantasy
D. at the time where they will explore their own genitals C. Formal operational stage
D. Concrete operational stage
66. Nurse Nina further assesses May. When BP was taken at 11:00 am, the reading A. An RR of 16 breaths per minute.
was at 140/80 mmHg. She knows that the physician may confirm that May is B. Urine output of 12 cc/hr. – UO = 30 cc/hr.
developing PIH when which criterion is satisfied? C. BP 110/80.
A. BP is at 130/90 mmHg at 12nn. D. DTR of 2+
B. A BP reading is read to be 140/90 at 1:30 pm 70. In case toxicity occurs, what emergency item must Nurse Nina prepare at bedside?
C. 30/15 mmHg is added to the baseline BP at 7:00 pm – 6hrs apart A. Protamine sulfate
D. BP is at 90/60 mmHg at 11:00 pm B. Atropine sulfate
67. May asks Nurse Nina to explain the reason why PIH occurs. Nurse Nina's most C. Kalcinate – Ca Gluconate
appropriate answer would be: D. Vitamin K
A. "An increase in blood volume to support the growing child caused an increased Situation: Health teaching is an essential nursing action to ensure the continuity of
pressure in the arteries causing damage to the walls. This led to the blood care beyond the borders of the institution. This helps promote independence to each
vessels in the body to suddenly tighten up or get narrower, thus causing the patient in managing their own health.
blood pressure to shoot up." 71. Nurse Kate teaches a preoperative patient about the nasogastric (NG) tube that
B. "Blood volume is increased by 40-50% in pregnancy, which usually peaks at will be inserted before surgery. She concludes that the patient understands when the
the 2nd trimester due to an increase in plasma volume. That caused an tube will be removed postoperatively based on which statement by the patient?
increase in pressure in the endothelial lining of the arteries Hypertension A. “When my doctor says so.”
occurs due to the narrowing of the blood vessels due to the spasms." B. “When I can tolerate food without vomiting.”
C. "The precise etiology revolves around aberrations in vascular homeostasis, C. “When my gastrointestinal (GI) system is healed.”
with dysregulation of endothelial function and vascular tone modulation being D. “When my bowels start working again and I begin to pass gas.” – good GI
paramount. This intricate cascade encompasses the intricate dance between functioning
vasoconstriction-promoting factors and vasodilation-mediating substances, 72. A patient is receiving intravenous lipid (fat emulsion) therapy at home, managed
leading to an imbalance favoring heightened vascular resistance and by the patient's spouse. During a visit, the Nurse Kate discusses the potential side
augmented systemic arterial pressure." effects and adverse reactions of the therapy with both the patient and the spouse. After
D. "I am not sure if I am allowed to give details about that diagnosis. I will ask your the discussion, she expects the spouse to understand that the priority action in case
physician to explain it to you instead." of a suspected adverse reaction is to:
68. The nurse further assesses May for any other signs while she interviews her in A. Stop the infusion.
the clinic as she documents the findings in May's chart. Which statement from May B. Contact the nurse.
would she record as possible evidence that May is developing pre-eclampsia instead? C. Take the patient’s blood pressure.
A. "I'm struggling to slip on my bedroom slippers because my feet are so puffy at D. Contact the local emergency response team.
night." 73. Nurse Kate suspects that a client's spouse may be experiencing caregiver strain.
B. "My mom taught me a lot of things about being pregnant, but she never warned Which action should the nurse take to assess this condition?
me that I would always feel so tired." A. Refer the family to a social services agency.
C. "I've been having the desire to really take care of how I look like these past few B. Gather information from both the caregiver and the client.
weeks." C. Wait for the caregiver to mention the stress.
D. "The puffy feet are tolerable, but the swollen hands and wrists are not.” D. Obtain feedback from the client about the caregiver.
69. Eventually, May's physician ordered magnesium sulfate to be infused on 74. Nurse Kate is educating a client newly diagnosed with diabetes mellitus about
piggyback with a loading dose of 5g to be infused slowly over 30 minutes and a blood glucose monitoring. The nurse should instruct the client to report glucose levels
maintaining dose of 2 g/hr. Which of the following symptoms, when observed by Nurse that consistently exceed which value?
Nina, would warrant her to discontinue the drug and notify the physician? A. 150 mg/dL (8.57 mmol/L)
B. 200 mg/dL (11.42 mmol/L) B. The client will report how to treat an infection.
C. 250 mg/dL (14.28 mmol/L) C. The client will be able to identify measures to prevent infection
D. 350 mg/dL (20.0 mmol/L) D. The client will identify the presence of Braxton Hicks contractions.
75. A client diagnosed with gastritis asks the Nurse Kate about analgesics that won't 79. Nurse Violet transfers to the NICU. She teaches handwashing techniques to the
cause epigastric distress. She should recommend which medication? parents of an infant who is receiving antibiotic treatment for a neonatal infection. She
A. Aspirin – further bleeding; GI irritant determines that the parents understand the primary purpose of handwashing if they
B. Naproxen – Nsaids make which statement?
C. Ibuprofen – Nsaids A. “It is primarily done to reduce their fears.”
D. Acetaminophen B. “It is primarily done to minimize the spread of infection to other siblings.”
Situation: It is fulfilling for Nurse Violet to be a maternal and child nurse. From health C. “It is primarily done to allow them an opportunity to communicate with each
education to emergency care interventions, in different stages of pregnancy, labor and other and staff.”
delivery, she performs her skills effectively. D. “It is primarily done to reduce the possibility of transmitting an environmental
76. When should Nurse Violet she plans to begin discharge planning to a mother with infection to the infant.”
a preterm infant? 80. She is preparing to educate a teenage client about sexuality. How will she start
A. When the mother is in labor the teaching?
B. When the discharge date is set A. Inform the teenager about the risks associated with pregnancy.
C. After the infant is stabilized during the early stages of hospitalization. B. Build a relationship and assess the teenager's existing knowledge.
D. When the parents feel comfortable with and can demonstrate adequate care C. Counsel the teenager to abstain from sexual activity until marriage.
for the infant D. Offer written materials concerning sexually transmitted infections.
77. Nurse Violet is performing an assessment on a primigravida client who has been Situation: Diabetes is one of the common, but also dangerous diseases in life. It can
a marathon runner for several years. The client verbalizes concern because she is no also be present among pregnant women.
longer able to run in marathons and is concerned about the brown discoloration on her 81. Charlotte is reviewing home care instructions with a client who has type 1 diabetes
face and her increasing size. Which statements by the Nurse Violet are therapeutic? mellitus and a history of diabetic ketoacidosis (DKA). The client's spouse is present
Select all that apply. during the instruction. Which statement by the spouse indicates that further teaching
I. “I can see you’re disappointed at not being able to run.” is needed? DKA = infection and lack of insulin
II. “Tell me how you are feeling about the changes in your body.” A. “If he is vomiting, I shouldn’t give him any insulin.”
III. “Don’t worry. Your body will go back to normal after delivery.” B. “I should bring him to the doctor if he develops a fever.”
IV. “You need to ask your obstetrician about whether or not you can run.” C. “If our grandchildren are sick, they probably shouldn’t come to visit.”
V. “Wait and see. You will be back to marathon running after delivery before you D. “I should call the doctor if he has nausea or abdominal pain lasting for more
know it.” than 1 or 2 days.”
VI. “Some of the changes in pregnancy are permanent and that is the price that 82. Nurse Charlotte has conducted a class for pregnant clients with diabetes mellitus
you have to pay for that bundle of joy.” about the signs and symptoms of potential complications. She determines that the
A. I, II teaching was effective if a client makes which statement?
B. II, III, IV A. “I should not have ultrasounds done because I am diabetic.”
C. I, III, V B. “I’m glad I don’t have to worry about developing hypoglycemia while I am
D. I, V, VI pregnant.”
78. Which goal is most appropriate for postpartum client who is at risk for uterine C. “I need to watch my weight for any sudden gains because I could develop
infection? gestational hypertension.”
A. The client will verbalize a reduction of pain.
D. “My insulin needs should decrease during the last 2 months because I will be 86. You are discussing contraceptive options with a 32-year-old patient and she
using some of the baby’s insulin supply.” expresses "I want to have children but not for a few years." You respond with the
DM 1 Glucose Insulin correct statement:
Pre-pregnancy Unstable Prescribed insulin A. "If you do not become pregnant within the next few years, you never will."
1st trimester ↓ = due to organogenesis ↓ B. "Women often have more difficulty becoming pregnant after about age 35."
2nd trimester ↑ = ↑ HPL ↑ C. "Stop taking oral contraceptives several years before you want to have a child."
3rd trimester ↑ ↑ D. "You have many more years of fertility left, so there is no rush to have children."
Post-term ↓ ↓ = drop rapidly 87. A 28-year-old patient reports experiencing anxiety, headaches with dizziness, and
83. Nurse Charlotte met with a postpartum patient who is recovering from abdominal bloating before her menstrual periods. What is the best course of action for
disseminated intravascular coagulation. They will need to take low dosages of you to take at this time?
anticoagulant medication. She advises them to avoid: A. Instruct the patient to track her symptoms in a diary for 3 months.
A. Brushing her teeth B. Recommend that the patient attempt aerobic exercise to alleviate her
B. Taking acetylsalicylic acid (aspirin) – risk for bleeding symptoms.
C. Walking long distances and climbing stairs C. Educate the patient about lifestyle adjustments to mitigate premenstrual
D. All activities due to the risk of bruising injuries syndrome (PMS) symptoms.
84. Nurse Charlotte is educating a mother diagnosed with diabetes mellitus who gave D. Counsel the patient to utilize nonsteroidal anti-inflammatory drugs (NSAIDs)
birth to a large-for-gestational-age (LGA) infant about infant care. She explains that like ibuprofen to manage symptoms.
LGA infants may appear more mature due to their size but often require stimulation for 88. A 19-year-old patient has received a diagnosis of primary dysmenorrhea. How will
feeding and attachment. Which statement by the mother suggests the need for further you advise the patient to address discomfort?
teaching about infant care? A. Refrain from engaging in aerobic exercise during menstruation.
A. “I will talk to my baby when he is in a quiet, alert state.” B. Apply cold packs to the abdomen and back for pain relief.
B. “I will allow my baby to sleep through the night because he needs his rest.” – C. Discuss with her healthcare provider the option of starting antidepressant
always stimulate the baby therapy.
C. “I will breast-feed my baby every 2½ to 3 hours and will use arousal D. Take nonsteroidal anti-inflammatory drugs (NSAIDs) at the onset of her period.
techniques.” 89. While attending to a 58-year-old patient experiencing persistent menorrhagia
D. “I will watch my baby closely because I know that he may not be as mature in (menorrhagia = excessive bleeding), you will intend to oversee the:
his motor development.” A. levels of estrogen.
85. She is observing a client with type 1 diabetes mellitus. Today's blood work reveals B. complete blood count (CBC) – check for anemia
a glycosylated hemoglobin level of 10%. Based on this result, she formulates a C. gonadotropin-releasing hormone (GNRH).
teaching plan, understanding that it signifies which outcome? D. consecutive human chorionic gonadotropin (HCG) outcomes.
A. A normal reading, indicating effective management of blood glucose control. 90. A 47-year-old patient inquires to you whether she is entering menopause after not
<6% having a menstrual period for 3 months. What would be a suitable response?
B. A reading devoid of information concerning the client's disease management. A. "Have you considered hormone replacement therapy?"
C. A low reading, indicating inadequate management of blood glucose control. B. "Many women experience some sadness when entering menopause."
D. A high reading, indicating inadequate management of blood glucose control. C. "What was your menstrual cycle like before your periods ceased?".
SITUATION: Good sexual and reproductive health is important for women's general D. "Given your age in the mid-40s, it's probable that you're experiencing
health and wellbeing. It is central to their ability to make choices and decisions about menopause."
their lives, including when, or whether, to consider having children.
Situation: Assessing the health of a newborn is very important for detecting any Situation: The Duchess of Hastings is one of the important clients in the Royal London
problems in their earliest, most treatable, stages. Nurse Colin is thorough yet quick to Hospital. She is seeking consultation for her current pregnancy.
make newborn assessments too be able to receive prompt treatments 96. At her first prenatal visit, a Daphne is being interviewed by the nurse for her health
91. A client has just delivered a 6-lb 2-oz boy. She is concerned about the way he history. She is pregnant for the fourth time, has had one baby at 38 weeks’ gestation,
looks. Nurse Colin, a delivery nurse reassures the mother that a pink body with purple premature twins at 35 weeks, and a spontaneous abortion. How would the nurse
feet and hands is a normal condition called: communicate this information to the healthcare team?
A. Acrocyanosis A. Gravida 4, para 3, a1
B. Mongolian condition – birthmarks that are present at birth and their most B. Gravida 4, para 2, T1, P2, A1, L3
common location is sacrococcygeal or lumbar area C. Gravida 4, para 3, T1, P2, A1, L3
C. Sternal retractions – RDS D. Gravida 4, para 2, T1, P1, A1, L3
D. Patent ductus arteriosus – a persistent opening between the two major blood 97. As part of a health history for pregnancy, she tells the nurse that she is unsure of
vessels leading from the heart. her blood type and Rh factor. She had a spontaneous abortion at 10 weeks and did
92. A baby’s chin is quivering and he is trembling a little postdelivery. Colin replies to not seek medical attention. After lab work, she is found to have type O, Rh-negative
the mother that this is probably due to a: blood. What high-risk factor should the nurse identify for the health team?
A. High sugar level – jittery = hypoglycemia A. The client may be at high risk for spontaneous abortion.
B. Very cold temperature – mottling B. The client may have become Rh sensitized during her first pregnancy.
C. Startle reflex C. The client’s fetus may have CNS malformations due to Rh incompatibility.
D. Immature nervous system D. The client is at risk for noncompliance with prenatal appointments.
93. A mother is unsure about breast-feeding her newborn. His best response would 98. Daphne further expresses that her LMP was June 10th. In order to determine her
be: estimated delivery date, what tool would the nurse use?
A. I’ll tell the nursery nurse that you want a bottle.” A. Naegele’s rule
B. “You have to decide immediately.” B. McDonald’s rule
C. “Let’s feed him by breast-feeding initially; I’ll help you.” C. Hegar’s sign
D. “Breast-feeding is best; don’t even consider bottle feeding.” D. Quickening
94. A mother took tetracycline during pregnancy for an acne condition. She asks if 99. She is about 3 weeks larger in uterine size than her dates indicate. The physician
there could be a problem with the baby. He responds that the baby could: orders an ultrasound to diagnose multiple fetuses. What information should the nurse
A. Be deaf give the client in preparation for the test?
B. Have discolored teeth – (+) bone formation A. “Do not eat or drink after midnight.”
C. Be a slow learner B. “Drink several glasses of water 1 hour before, and do not urinate.” – stabilize
D. Have webbed fingers and toes uterus
95. A couple comes to the clinic very excited because the woman missed her period C. “Empty your bladder immediately before the test.” – amniocentesis; Leopold’s
5 weeks ago. She is experiencing early morning nausea, urinary frequency, and maneuver
fatigue. In anticipating questions about pregnancy, he would need to be aware that D. “Give yourself an enema prior to the test.”
these are: 100. While assessing Daphne, another client is wheeled in at the ER. She is at 35
A. Positive signs of pregnancy weeks’ gestation in labor with a diagnosis of abruptio placenta (use of cocaine). What
B. Probable signs of pregnancy pattern on the fetal monitor should the nurse anticipate?
C. Presumptive signs of pregnancy A. Late decelerations
D. Signs that also could indicate bladder infection B. Early decelerations
C. Variable decelerations
D. Accelerations with fetal movement 5. A client with atrial fibrillation is being taught by the nurse the importance of starting
COMPREHENSIVE PHASE - RECALLS 1 (NP3) long-term anticoagulant medication. Which justification for this therapy works best,
Situation: NCDs in the Philippines are causing a surge in health-care costs and social according to the nurse?
care and welfare support needs and are contributing to reduced productivity. A. "This dysrhythmia causes blood to back up in the legs, which increases the risk
1. Jill, the head nurse of Ward 1, is supervising the new hired nurse as they auscultate of blood clots."
a client's breath sounds. There is a need to intervene when nurse Jill observes the B. “Blood clots may form in the brain as a result of this dysrhythmia, which lowers
new nurse to? the amount of blood pumping from the heart.”
A. Utilizing the bell of the stethoscope – bell is for low pitched sounds; breath C. "You need this medication to prevent blood clots, which are a side effect of the
sounds = use diaphragm antidysrhythmic medications you are taking."
B. Requesting the client to sit upright D. “Blood travels slowly through the quivering atria, which can cause clots to form
C. Positioning the stethoscope directly on the client's skin along the heart wall. These clots may then release and move to other parts of
D. Advising the client to inhale slowly and deeply through the mouth the body such as the brain or lungs.”
2. Digoxin is used to improve the strength and efficiency of the heart, or to control the Situation: In the Philippines, a Southeast Asian nation of over 110 million people,
rate and rhythm of the heartbeat. This leads to better blood circulation and reduced cancer is amongst the leading causes of death.
swelling of the hands and ankles in patients with heart problems. The nurse instructs 6. You are making a discharge plan for a patient with leukemia. Instructions should
the parents on how to give digoxin. Which of the mother's statements implies that more include:
teaching is necessary? A. Monitor rectal temperatures every 4 hours.
A. "I'll combine the medicine with food." – digoxin is taken without food B. Check the mouth and anus after every shift for any tissue breakdown.
B. "Before giving my child the medication, I will check their pulse." C. To preserve the child's nutritional health, encourage them to eat fresh fruits and
C. "I won't give the medication again if my child throws up after I give it to them." vegetables.
D. "Before giving my husband his medication, I will talk to him about the dosage.” D. Use a toothbrush and mouthwash with alcohol multiple times a day to give your
3. There is a high risk for the development of heart failure among patients with valvular mouth the careful attention it deserves.
heart disease. What should the nurse prioritize when monitoring for signs of heart LEUKEMIA
failure? ✓ Thrombocytopenia
A. Pulse rate ✓ Anemia
B. Breath sounds – crackles = increase blood volume ✓ leukocytopenia
C. Blood pressure 7. A patient asks you what makes the malignant tumors different from benign tumors.
D. Ability to tolerate activity What is your response?
4. Nurse Karina provides discharge instructions to mother whose child just underwent A. “Benign tumors do not cause damage to other tissues.”
heart surgery. Which instructions should she give the mother? B. “Benign tumors are likely to recur in the same location.” – both recurs
A. The child is allowed to play outside for brief intervals. – play outside for several C. “Malignant tumors may spread to other tissues or organs.” – metastasis
weeks D. “Malignant cells reproduce more rapidly than normal cells.”
B. After taking a bath, apply powder and lotion to the incision. – no powder and 8. You are caring for a patient who smokes two packs/day. What can you do to help
lotion lessen the risks of developing cancer?
C. After leaving the hospital, the child has one week to go back to school. – 3 A. Teach the patient about the seven warning signs of cancer.
weeks after discharge = school B. Plan to monitor the patient’s carcinoembryonic antigen (CEA) level.
D. In the event that the child has a temperature greater than above 100.5°F, or C. Teach the patient about annual chest x-rays for lung cancer screening.
38°C, she must notify the physician D. Discuss risks associated with cigarette smoking during each patient encounter
– biggest risk factor for cancer
9. The nurse should suggest which food choice when providing dietary teaching for a 13. Kira is reviewing the electrocardiogram (ECG) rhythm strip of a client with a history
patient scheduled to receive external-beam radiation for abdominal cancer? of a myocardial infarction (MI) notes that the PR intervals are 0.16 seconds. Kira
A. Fruit salad interprets this as?
B. Creamy mushroom soup A. A normal finding – PR = 0.12-0.20 secs
C. Roasted chicken – increase protein B. An abnormal finding
D. Toasted wheat bread C. An impending reinfarction
10. You are teaching a postmenopausal patient with stage III breast cancer about the D. First-degree atrioventricular (AV) block
expected outcomes of cancer treatment. Your teaching is effective when the patient 14. A code is called on the unit. Kira is administering emergency care to a patient
states: experiencing ventricular tachycardia is getting ready to perform defibrillation. What
A. “After cancer has not recurred for 5 years, it is considered cured.” nursing action ensures the safest environment during this procedure?
B. “The cancer will be cured if the entire tumor is surgically removed.” A. Confirming the absence of any lubricant on the paddles – presence
C. “I will need follow-up examinations for many years after treatment before I can B. Applying the charged paddles one by one to the patient's chest – simultaneous
be considered cured.” – possible recurrence C. Stabilizing the patient's upper torso while defibrillation is carried out
D. “Cancer is never cured, but the tumor can be controlled with surgery, D. Ensuring that all assisting staff are clear of the patient and the patient's bed.
chemotherapy, and radiation.” 15. Her patient is experiencing unstable ventricular tachycardia (VT) loses
Situation: Acute Myocardial Infarction, which includes ST-Elevation Myocardial consciousness and becomes pulseless following an initial intravenous dose of
Infarction (STEMI), has the highest rate of mortality among the cardiovascular lidocaine. What should be the next equipment she should get?
diseases with an average mortality rate of 10% based on health facility records in the A. A pacemaker
Philippines. Nurse Kira is a dedicated nurse who specializes in cardiovascular health. B. A defibrillator – if (-) defib = CPR
11. Nurse Kira is handling a lot of patients. By her confusion, Kira administers digoxin C. A second dose of lidocaine
0.25 mg by mouth rather than the prescribed dose of 0.125 mg to the client. After D. An electrocardiogram machine
assessing the client and notifying the health care provider, which action should Kira Situation: Nurse Nikki handles multiple patients in the medicine ward.
implement first? 16. Nurse Nikki monitors a patient receiving continuous intravenous infusion of
A. Write an incident report. heparin sodium therapy. What is the adverse effect she should watch out for?
B. Administer digoxin immune Fab. – digoxin toxicity A. Tinnitus
C. Tell the client about the medication error. B. Ecchymoses – signs of bleeding
D. Tell the client about the adverse effects of digoxin. C. Increased pulse rate
12. Kira’s patient is ordered for nitroglycerin transdermal medication patches. What D. Decreased blood pressure
are the lessons included for teaching? Select all that apply. 17. She noticed her patient has an activated partial thromboplastin time (aPTT) value
I. Apply a new medication patch every 7 days. – daily for 12-16 hours of 100 seconds. Before reporting the results to the primary health care provider, she
II. Apply a new medication patch in the morning. verifies that which medication is available for use if prescribed?
III. Keep a patch in place for 12 to 16 hours or as directed. A. Vitamin K – warfarin tox.
IV. Wait 1 day to apply a new medication patch if it becomes dislodged. B. Vitamin B12 – pernicious anemia
V. Place the medication patch in the area of a skinfold to promote better C. Methylene blue – cyanide poisoning
adherence D. Protamine sulfate -
A. I, III, IV 18. She is called to the ER for an emergency. A patient arrives at the emergency
B. II, III department with severe, radiating chest pain, exhibiting extreme restlessness, fear,
C. II, IV, V and shortness of breath. Immediate admission orders include administering oxygen
D. I, II via nasal cannula at 4 L per minute, obtaining troponin, creatinine phosphokinase, and
isoenzymes blood levels, performing a chest x-ray, and conducting a 12-lead ECG. D. perform a neurologic assessment to establish a baseline.
What should she prioritize? 23. A patient with cardiogenic shock receives a nursing diagnosis of decreased
A. Conducting the 12-lead ECG. cardiac output. With the appropriate interventions, the anticipated outcome is for the
B. Collecting blood specimens. patient to achieve:
C. Administering oxygen to the patient. – prevent further cardiac damage A. baseline activity level.
D. Scheduling the chest x-ray examination. B. baseline cardiac function.
19. A patient experiences an irregular heart rate. Which statement from the patient C. decreased afterload.
suggests to the nurse that they are prepared to learn about their condition? D. reduced anxiety.
A. "I feel weak with an irregular pulse." 24. A patient recently had a cardiac catheterization via right-radial approach and now
B. "What are the effects of having a pacemaker?" has a compression device in place. The patient reports numbness and pain in the right
C. "Will all my medications be altered?" hand. The cardiac-vascular nurse notes a diminished pulse, with a cool and cyanotic
D. "How might this heart rate issue impact me?" hand. The nurse:
20. The cardiac-vascular nurse reviews risk factor reduction with a patient who is A. calls the physician.
newly diagnosed with a myocardial infarction. The patient states, "I don't know why B. performs an Allen test.
you're making such a big deal about this stuff. I feel fine, and the doctor said that my C. reduces the pressure on the puncture site. – possible poor circulation
heart attack was small." The nurse's most effective action is to: D. uses the Doppler ultrasound to assess for pulse signals.
A. assess the patient's perception of the event with open-ended questions 25. In an assessment for intermittent claudication, the cardiac-vascular nurse
B. present research to support the need for risk-factor reduction. assesses for leg pain and cramping with exertion, then asks the patient,
C. reinforce patient education. A. "Does shortness of breath accompany the leg pain?"
D. review the laboratory values with the patient. B. "Does this same type of pain occur without activity?"
Situation: Working in the catheterization laboratory involves many specialized and C. "Is the leg pain relieved by rest?" – sign of arterial insufficiency
high nursing skills. D. "Is the leg pain relieved with elevation?"
21. While the cardiac-vascular nurse preceptor is orienting a graduate nurse on the Situation: Oxygen treatment and management is essential in every nursing care.
telemetry unit, a patient experiences cardiac arrest. Which action by the preceptor, 26. Patients in mechanical ventilation are prone to infection. What measures should
during the emergency cardiac care procedure, facilitates the graduate nurse's the nurse take to avoid ventilator-associated pneumonia (VAP) in intubated clients
competence and professional development? receiving mechanical ventilation?
A. Asking the graduate nurse to review the policy and procedure for cardiac A. Adhere to thorough hand hygiene protocols.
arrest. B. Keep the head of the bed elevated at a 10-degree angle.
B. Assigning the graduate nurse to comfort the family during the arrest. C. Conduct oral cavity secretion suctioning every 4 hours.
C. Directing the graduate nurse to attempt IV access. D. Ensure the respiratory therapist replaces the ventilator circuit tubing every 4
D. Involving the graduate nurse in the resuscitation by assigning a basic task. hours.
22. A patient is admitted to the hospital for a carotid angiogram with stent placement. 27. Which ordered intervention should the nurse refrain from performing until a
The patient's spouse states, "I don't want my spouse to find out there is a risk of a comatose patient is appropriately intubated?
stroke connected with this procedure because they won't sign the consent form." The A. Administering gastric feeding – risk for aspiration
cardiac-vascular nurse's most appropriate action is to: B. Inserting a urethral catheter
A. assess the patient's level of understanding of the risks, benefits, and C. Conducting a finger stick to assess blood glucose level
alternatives. D. Performing venipuncture for a complete blood cell (CBC) count
B. assure the patient's spouse that the risk of stroke is minimal.
C. offer the patient emotional support and reinforce the benefits of the procedure.
28. A patient diagnosed with active tuberculosis (TB) is scheduled for admission to a 32. She is caring for a client with chronic obstructive pulmonary disease (COPD). They
medical-surgical unit. What action should the nurse prioritize when arranging a bed are receiving home oxygen therapy at a rate of 2 liters per minute. She notes the
assignment? client's respiratory rate as 22 breaths per minute. When the client complains of
A. Assigning the patient to a private, well-ventilated room. – (-) pressure worsening dyspnea, what should the nurse do first?
B. Arranging for the patient's transfer to the intensive care unit. A. Assess whether there is a need to adjust the oxygen flow.
C. Allocating the bed farthest from the door in a double room. B. Contact emergency services for assistance at the residence.
D. Placing the patient in a shared double room with a noninfectious patient. C. Offer reassurance to the client, emphasizing there is no cause for concern.
29. The nurse has completed suctioning a client's tracheostomy. What should the D. Gather additional details regarding the client's respiratory condition.
nurse monitor to assess the procedure's effectiveness? 33. She is providing discharge instructions to the mother of an 8-year-old child who
A. Auscultation of breath sounds had a tonsillectomy. The mother tells her that the child loves tacos and asks when the
B. Capillary refill time child can safely eat one. To prevent complications of the surgical procedure, what
C. Respiratory rate should be the appropriate response to the mother? – avoid red liquids (confuse for
D. Oxygen saturation levels bleeding); spicy and scratchy food (irritant)
30. The nurse is ready to initiate emergency care for a patient exhibiting signs of a A. “In 1 week.”
pulmonary embolism. Which instruction from the primary healthcare provider should B. “In 3 weeks.”
the nurse prioritize? C. “Six days after surgery.”
A. Administer oxygen. D. “When the primary health care provider says it is okay.”
B. Provide morphine sulfate. 34. There is a need for nurse Vanessa to reinforce incentive spirometry teaching if the
C. Initiate an intravenous (IV) line. client states that they should:
D. Perform an electrocardiogram (ECG). A. Inhales slowly
Situation: Respiratory nurses work closely with patients of all ages to address an B. Breathes through the nose
array of respiratory health issues. Through patient assessment, physical C. Removes the mouthpiece to exhale
examinations, reviewing patient medical histories, monitoring and recording vital signs, D. Forms a tight seal around the mouthpiece with the lips
and discussing symptoms, nurses in this specialty can treat and cure a myriad of Situation: Diabetes continues to be the fourth leading cause of mortality in the
conditions. Philippines. Chronic complications are very debilitating with far-reaching
31. Vanessa, a respiratory nurse, is caring for a patient who will undergo a consequences.
bronchoscopy. Which actions should she incorporate into the care plan for a patient 35. You are monitoring a client for a hypoglycemic reaction, knowing that NPH insulin
scheduled for a bronchoscopy? Choose all that apply. peaks in approximately how many hours following administration?
I. Remove dentures, if present. A. 1 hour
II. Remove contact lenses, if worn. B. 2 to 3 hours
III. Limit access to food and drink. C. 4 to 12 hours.
IV. Confirm that informed consent has been obtained. D. 16 to 24 hours
V. Ensure the patient empties their bladder before being transported for the 36. A client diagnosed with type 2 diabetes mellitus is being discharged from the
procedure. hospital after an occurrence of hyperglycemic hyperosmolar state (HHS). You are
A. I, II, III planning to create a discharge teaching plan for the client and identifies which
B. I, III, V intervention as a priority?
C. II, IV, V A. Exercise routines
D. I, II, IV, V B. Controlling dietary intake
C. Keeping follow-up appointments
D. Monitoring for signs/symptoms of dehydration D. II, III, V
37. You create a plan of care for an older client diagnosed with diabetes mellitus. It is 41. The nurse is giving epoetin alfa to a client with chronic kidney disease (CKD). What
important that you plan to complete which action first? adverse effect of this treatment should the nurse watch for in the client?
A. Structure menus for adherence to diet. A. Anemia
B. Teach with videotapes showing insulin administration to ensure competence. B. Hypertension – increase risk for hypertension and seizure activities
C. Encourage dependence on others to prepare the client for the chronicity of the C. Iron overload
disease. D. Increased bleeding risk
D. Assess the client’s ability to read label markings on syringes and blood glucose 42. A client diagnosed with chronic kidney disease (CKD) has received dietary advice
monitoring equipment. regarding potassium restriction. The nurse confirms the client has understood the
Situation: On World Kidney Day, observed on March 14, 2024 the Philippine Society information when they indicate they will take what action when preparing vegetables?
of Nephrology (PSN) is organizing a lay forum dedicated to educating Filipino on CKD A. Consume only fresh vegetables.
with the theme of “Kidney Health for All: Advancing Equitable Access to Care and B. Boil them and discard the cooking water.
Optimal Medication Practice.” The primary objective is to raise awareness about the C. Exclusively use salt substitutes for seasoning. – high in potassium = avoid salt
significance of ensuring equal access to appropriate treatment and care for individuals substitutes
living with kidney disease. This, in turn, aims to enhance their quality of life and delay D. opt for frozen vegetables whenever feasible.
the progression of the disease. 43. Why is diphenhydramine given before a blood transfusion?
38. While observing a patient diagnosed with chronic kidney disease (CKD), which A. To prevent hives.
observation warrants reporting to the primary healthcare provider? B. To prevent fever and chills
A. Pallor C. To improve clotting factors
B. Fatigue signs of anemia D. To increase blood volume
C. Lethargy 44. The nurse reviewing a urinalysis report for a client with the diagnosis of acute
D. Petechiae. – abnormal bleeding kidney injury notes that the results are highly positive for proteinuria. The nurse
39. The nurse is overseeing a client diagnosed with hypercalcemia. What observation determines that this client has which type of renal failure?
suggests a requirement for further evaluation? A. Prerenal failure
A. Elevated peristalsis B. Postrenal failure
B. Diminished capillary refill. (+) of clots C. Intrinsic renal failure – proteinuria = damaged kidneys
C. Heightened deep tendon reflexes D. Atypical renal failure
D. Reduced abdominal circumference Situation: Kovu, a 6-year-old male, suddenly presents with excessive swelling. He is
40. A patient diagnosed with acute kidney injury has elevated blood urea nitrogen diagnosed with nephrotic syndrome.
(BUN) levels and is struggling with memory retention. What interventions should the 45. In your nursing care plan, you will include which of the following as a nursing
nurse employ when communicating with this patient? Select all that apply. diagnosis for this patient?
I. Provide straightforward, understandable instructions. A. Risk for infection – decrease proteins =low immunoglobulin = DOC: steroids =
II. Involve the family in care-related discussions. low immunity
III. Describe treatments using plain language. B. Deficient fluid volume
IV. Avoid providing extensive, detailed explanations of procedures. C. Constipation
V. Utilize various teaching methods to deliver discharge instructions. D. Overflow urinary incontinence
A. I, IV, V 46. You know that you should assess which important parameter on a daily basis?
B. I, II, III A. Weight – edema/fluid retention
C. I, III, V B. albumin levels
C. activity tolerance breath sounds, and dullness to percussion on the right lower lung field. The nurse
D. blood urea nitrogen (BUN) level suspects a complication of pneumonia that involves accumulation of fluid in the pleural
47. You need to collect urine sample. You know that the urine sample will appear: space. Which of the following conditions is the nurse likely suspecting?
A. Tea-colored – glomerulonephritis a) Empyema – pus in the pleural cavity = types: 1) transudate and 2) exudate
B. Orange and frothy – orange = rifampicin b) Pleural effusion
C. Dark and foamy – proteinuria c) Atelectasis
D. Straw-colored d) Pulmonary embolism – blockage of pulmonary artery by a blood clot =
48. Which assessment finding below requires you to notify the physician immediately decrease blood flow to the lungs
for a patient with nephrotic syndrome? 53. Mr. Johnson is at risk for the development of atelectasis due to his immobility and
A. Frothy, dark urine shallow breathing. Which of the following nursing interventions is most appropriate to
B. Redden area on the patient's left leg that is swollen and warm – infection prevent atelectasis?
C. Elevated lipid level on morning labs a) Encourage deep breathing and coughing exercises – expand the lungs and
D. Urine test results that show proteinuria clear secretion = lung expansion
49. It has been identified that Kovu has acute glomerulonephritis. During history-taking b) Administer bronchodilator medications
the you first ask Kovu’s mother about a recent history of: c) Apply oxygen via nasal cannula
A. bleeding ulcer d) Perform chest physiotherapy
B. deep vein thrombosis 54. Mr. Johnson's condition deteriorates, and he develops sudden chest pain and
C. myocardial infarction dyspnea. The nurse suspects a possible pneumothorax. Which of the following
D. streptococcal infection – strep throat/ sre throat findings would support this suspicion?
50. The client with acute renal failure has a serum potassium of 6.0 mEq/L. The nurse a) Hyperresonance on percussion – air filled space
would plan which of the following as a priority action? b) Decreased tactile fremitus – air/fluid in pleural space or decrease in lung
A. check the sodium level density (pulmonary emphysema, COPD, asthma)
B. place the client on a cardiac monitor – hyperkalemia = ECG = tall peaked T ✓ Tactile Fremitus = the palpable vibration of the chest wall that results from the
wave transmission of sound vibrations through the lung tissue to the chest wall
C. encourage increased vegetables in the diet c) Crackles on auscultation – crackles = AKA: rales
D. allow an extra 500 ml of fluid intake to dilute the electrolyte concentration d) Increased vocal resonance – lung tissue become denser than normal (lung
Scenario: Mr. Johnson, a 65-year-old patient, is admitted to the hospital with a tumor and atelectasis)
diagnosis of pneumonia. As the nurse, you are responsible for his care and monitoring 55. After confirming a pneumothorax, the nurse anticipates the need for intervention
his respiratory status. to remove the air from the pleural space. Which of the following interventions is the
51. While assessing Mr. Johnson, you note that he has increased respiratory rate, primary treatment for a pneumothorax?
shallow breathing, and decreased breath sounds on auscultation. Which of the a) Chest tube insertion
following conditions should you suspect? b) Administration of antibiotics
a) Atelectasis – collapse or incomplete expansion of lung tissue c) High-flow oxygen therapy
b) Pneumothorax – air accumulation in the lungs d) Percutaneous needle aspiration
c) Pulmonary embolism – tachycardia, pleuritic chest pain, signs of DVT 56. Mr. Johnson develops sudden dyspnea, tachycardia, and pleuritic chest pain. The
d) Acute respiratory distress syndrome (ARDS) – severe form of respiratory nurse suspects a pulmonary embolism. Which of the following diagnostic tests would
failure be most appropriate to confirm this suspicion?
52. A 40-year-old female patient is admitted to the hospital with a diagnosis of a) Chest X-ray
pneumonia. Upon assessment, the nurse notes increased respiratory rate, decreased b) Electrocardiogram (ECG)
c) Pulmonary angiography – gold standard; diagnose for PE; injecting contrast d) BNP acts as a vasodilator. – promotes vasodilation ad natriuresis; BNP =
dye to pulmonary artery relaxed in response to release of increase wall stress and volume overload
d) D-dimer blood test – checks blood clotting protein; if (+) = blood clots 61. Mrs. Anderson has a history of hypertension and is prescribed an angiotensin-
Scenario: Mrs. Anderson, a 65-year-old patient, is admitted to the hospital with a converting enzyme (ACE) inhibitor for heart failure management. Which of the
diagnosis of heart failure. As the nurse, you are responsible for her care and monitoring following is the primary action of ACE inhibitors in heart failure?
her cardiovascular status. a) Dilating coronary arteries
57. Mrs. Anderson complains of shortness of breath and fatigue upon exertion. Which b) Reducing systemic vascular resistance
of the following assessments is most important for the nurse to perform to evaluate c) Enhancing myocardial contractility
her heart failure status? d) Increasing sodium and water retention
a) Blood pressure measurement
b) Respiratory rate assessment
c) Electrocardiogram (ECG) monitoring
d) Daily weight measurement – HF = ineffective pumping ability of the heart =
fluid build up in the body = weight gain; detect early sign of worsening heart
failure
58. Mrs. Anderson is prescribed furosemide (Lasix) for the management of her heart
failure. Which of the following is the primary action of furosemide? (loop diuretics)
a) Reducing preload – amount of blood already in heart before pumping
b) Increasing afterload – pressure against which the heart has to pump blood out
of the systemic circulation
c) Enhancing myocardial contractility
d) Dilating coronary arteries
59. Ms. Johnson, a 68-year-old patient, is admitted to the hospital with a diagnosis of
left-sided heart failure. The nurse assesses crackles in the patient's lung bases and
reports this finding to the healthcare provider. Which of the following nursing Scenario: Mr. Johnson, a 55-year-old patient, is admitted to the hospital with a
interventions would be most appropriate to manage the patient's respiratory diagnosis of acute myocardial infarction (AMI). As the nurse, you are responsible for
symptoms? his care and monitoring his cardiac status.
a) Administering supplemental oxygen – ineffective ability to pump blood = fluid 62. Mr. Johnson complains of severe chest pain radiating to his left arm and jaw. The
buildup = pulmonary edema= (+) crackles = respiratory distress nurse suspects myocardial ischemia. Which of the following laboratory tests is most
b) Assisting with chest physiotherapy commonly used to diagnose AMI?
c) Encouraging deep breathing exercises a) Troponin – gold standard; increase levels of troponin = myocardial cell death
d) Administer diuretics =rises within 3-4 hours of the onset of chest pain; remains elevated for 1-
60. Mrs. Anderson's laboratory results show elevated levels of brain natriuretic peptide 2weeks
(BNP). The nurse recognizes this as a diagnostic marker for heart failure. Which of the b) C-reactive protein (CRP) – inflammatory marker
following statements best explains the role of BNP in heart failure? c) Brain natriuretic peptide (BNP) – more indicative of HF rather than MI
a) BNP causes peripheral vasoconstriction. d) Creatine kinase (CK) – lack specificity; present in various tissues
b) BNP increases myocardial contractility. 63. Mr. Johnson is prescribed aspirin as part of his treatment plan for AMI. Which of
c) BNP promotes fluid retention. the following is the primary action of aspirin in this context?
a) Inhibiting platelet aggregation
b) Reducing myocardial oxygen demand 68. Mr. Thompson is prescribed digoxin as part of his heart failure management. The
c) Dilating coronary arteries nurse understands that digoxin primarily works by:
d) Enhancing myocardial contractility a) Enhancing myocardial contractility – digoxin = cardiac glycoside/ cardiotonic;
ASPIRIN 4As + ino; - chrono
✓ Antiplatelet b) Dilating coronary arteries
✓ Antipyretic c) Reducing systemic vascular resistance – ACE inhibitor
✓ Analgesic d) Inhibiting platelet aggregation – anticoagulants
✓ Anti-inflammatory INOTROPIC CHRONOTRPIC
64. Mr. Johnson undergoes percutaneous coronary intervention (PCI) to restore blood ✓ Affect, cardiac contraction ✓ Affect HR
flow to the affected coronary artery. Which of the following nursing interventions is
most important immediately after PCI? 69. Mr. Thompson's blood work reveals low levels of potassium (hypokalemia). The
a) Monitoring vital signs every 4 hours nurse understands that this is a potential side effect of which medication commonly
b) Administering anticoagulant medications used in heart failure?
c) Assessing the puncture site for bleeding – risk for bleeding, hematoma a) Furosemide (Lasix) – loop diuretic; K-wasting
d) Encouraging early ambulation b) Lisinopril (Zestril) – ACE inhibitor
65. Mr. Johnson is prescribed nitroglycerin for the relief of angina symptoms. Which c) Metoprolol (Lopressor)
of the following statements best explains the action of nitroglycerin in angina? d) Spironolactone (Aldactone) – K-sparing
a) Nitroglycerin dilates coronary arteries. ↑ blood flow and O2 supply = relieving 70. Mr. Thompson is educated on dietary modifications to manage his heart failure.
Sx of angina Which of the following foods should the nurse advise him to limit in his diet?
b) Nitroglycerin reduces myocardial oxygen demand. – reducing preload a) Lean protein sources
c) Nitroglycerin enhances myocardial contractility. b) Fresh fruits and vegetables
d) Nitroglycerin inhibits platelet aggregation. c) Whole grains
66. Mr. Johnson is at risk for developing heart failure following his AMI. Which of the d) Sodium-rich foods.
following manifestations would the nurse expect to assess in a patient with heart Scenario: A 65-year-old patient presents with shortness of breath and a persistent
failure? cough. Upon auscultation, the nurse hears crackles in the lung fields. The patient also
a) Bradycardia and hypotension reports orthopnea. The nurse suspects a respiratory condition and performs a
b) Jugular vein distention and peripheral edema comprehensive respiratory assessment.
c) Increased urine output and weight loss 71. Which structure warms and filters inspired air in the upper respiratory tract?
d) Hyperactive bowel sounds and diarrhea a) Trachea
Scenario: Mr. Thompson, a 70-year-old patient, is admitted to the hospital with b) Nasal cavity
worsening heart failure. As the nurse, you are responsible for his care and c) Bronchi
management of his symptoms. d) Alveoli
67. Mr. Thompson presents with dyspnea at rest, orthopnea, and paroxysmal 72. What is the primary function of the paranasal sinuses?
nocturnal dyspnea. The nurse recognizes these symptoms as indicative of: a) Resonating chamber in speech
a) Left-sided heart failure – pulmonary symptoms b) Gas exchange
b) Right-sided heart failure – systemic symptoms c) Production of mucus
c) Diastolic heart failure d) Airway protection
d) Systolic heart failure 73. Which structure covers the opening to the larynx during swallowing?
a) Epiglottis
b) Glottis c) Whispered sounds heard loudly and clearly upon auscultation
c) Vocal cords d) Vibrations of speech felt as tremors of the chest wall during palpation
d) Thyroid cartilage Scenario: A 50-year-old patient presents with a persistent cough and hemoptysis. The
74. What is the primary function of the trachea? nurse suspects a respiratory condition and performs a respiratory assessment.
a) Gas exchange 81. What is the term for expectoration of blood from the respiratory tract?
b) Vocalization a) Apnea
c) Airway protection b) Rhonchi
d) Passage of air between larynx and bronchi c) Hemoptysis
75. Which structure is responsible for gas exchange in the lower respiratory tract? d) Tachypnea
a) Alveoli 82. Which respiratory assessment finding is associated with stridor?
b) Trachea a) Harsh high-pitched sound on inspiration.
c) Bronchi b) Low-pitched wheezing or snoring sound
d) Larynx c) Soft, high-pitched popping sounds during inspiration
Scenario: A 45-year-old patient presents with a persistent cough and wheezing. The d) Abnormal increase in clarity of transmitted voice sounds
nurse suspects airway narrowing and obstruction and performs a respiratory 83. Which respiratory assessment finding is associated with wheezes?
assessment. a) Harsh high-pitched sound on inspiration – expiration
76. Which respiratory assessment finding is associated with crackles? b) Low-pitched wheezing or snoring sound
a) Harsh high-pitched sound on inspiration – stridor c) Soft, high-pitched popping sounds during inspiration
b) Low-pitched wheezing or snoring sound – rhonchi d) Abnormal increase in clarity of transmitted voice sounds
c) Soft, high-pitched popping sounds during inspiration. 84. What is the term for a decrease in arterial oxygen tension in the blood?
d) Abnormal increase in clarity of transmitted voice sounds – bronchophony a) Hypoxia
77. Which respiratory assessment finding is associated with rhonchi? b) Orthopnea
a) Harsh high-pitched sound on inspiration c) Hypoxemia.
b) Soft, high-pitched popping sounds during inspiration d) Tidal volume
c) Low-pitched wheezing or snoring sound 85. What is the term for a decrease in oxygen supply to the tissues and cells?
d) Abnormal increase in clarity of transmitted voice sounds a) Hypoxia
78. What is the term for a subjective experience that describes difficulty breathing? b) Orthopnea
a) Dyspnea c) Hypoxemia
b) Apnea d) Tidal volume
c) Tachypnea Scenario: A 30-year-old patient presents with excessive daytime sleepiness and a
d) Orthopnea history of interrupted breathing during sleep. The nurse suspects a respiratory
79. Which respiratory assessment finding is associated with egophony? condition and performs a respiratory assessment.
a) Harsh high-pitched sound on inspiration – stridor 86. What is the term for the temporary absence of breathing during sleep due to
b) Soft, high-pitched popping sounds during inspiration – crackles transient upper airway obstruction?
c) Whispered sounds heard loudly and clearly upon auscultation a) Sleep apnea
d) Vibrations of speech felt as tremors of the chest wall during palpation – fremitus b) Apnea
80. Which respiratory assessment finding is associated with fremitus? c) Orthopnea
a) Harsh high-pitched sound on inspiration d) Dyspnea
b) Soft, high-pitched popping sounds during inspiration
87. What is the term for shortness of breath when lying flat, relieved by sitting or <3 years old Adult
standing? ✓ Down and back ✓ Up and back
a) Hypoxemia 93. A nurse is assessing the temperature of a 6-year-old child. The nurse wants to
b) Orthopnea use a method that is preferred for this age group. Which site should the nurse use to
c) Tachypnea obtain a temperature reading in this child?
d) Hemoptysis a) Oral
88. What is the term for the percentage of hemoglobin that is bound to oxygen? b) Tympanic
a) Oxygen saturation c) Rectal
b) Compliance d) Axillary
c) Physiologic dead space 94. A nurse is assessing the temperature of an older adult. The nurse knows that
d) Pulmonary diffusion older adults' temperatures tend to be lower than those of middle-aged adults. Which
89. What is the term for the portion of the tracheobronchial tree that does not factor contributes to this difference?
participate in gas exchange? a) Increased environmental temperature
a) Compliance b) Increased metabolic rate – decrease
b) Physiologic dead space c) Decreased efficiency of thermoregulation control processes.
c) Pulmonary perfusion d) Increased cardiac output – decrease
d) Respiration 95. A nurse is assessing a patient's vital signs and notices an elevated pulse rate. The
90. What is the term for gas exchange between atmospheric air and the blood, and nurse understands that various factors can influence the pulse rate. Which of the
between the blood and cells of the body? following factors is most likely responsible for the increased pulse rate in this patient?
a) Compliance a) Anxiety
b) Physiologic dead space b) Hypothermia
c) Pulmonary perfusion c) Hypoglycemia
d) Respiration d) Dehydration
91. A nurse is assessing a newborn infant's temperature. The nurse knows that 96. A nurse is performing a temperature assessment on a 2-year-old child. The nurse
newborns are at risk for hypothermia and need to be kept warm and dry to maintain wants to use a noninvasive method that is suitable for this age group. Which site
their body temperature. Which site should the nurse use to obtain an accurate should the nurse use to obtain a temperature reading in this child?
temperature reading in a newborn? a) Oral
a) Oral b) Tympanic
b) Tympanic c) Rectal
c) Rectal. d) Axillary
d) Axillary 97. A nurse is assessing the temperature of an older adult. The nurse notices that the
92. A nurse is assessing a 3-month-old infant's temperature. The nurse wants to use client has a buildup of ear cerumen (earwax). How can this affect the accuracy of
a method that is fast and convenient. Which site should the nurse use to obtain a tympanic thermometer readings in older adults?
temperature reading in this infant? a) It may cause a falsely elevated temperature reading.
a) Oral b) It may cause a falsely decreased temperature reading – block infrared waves
b) Tympanic c) It may have no effect on the temperature reading.
c) Rectal d) It may cause discomfort to the client.
d) Axillary
98. A nurse is assessing the temperature of a client and notes that the client has D. Increased respiratory rate
hemorrhoids. How should the nurse proceed when obtaining a rectal temperature 3. The nurse is reviewing the laboratory results of a patient's nutritional assessment.
reading in this client? Which laboratory test provides an estimate of visceral protein stores?
a) Insert the thermometer gently to avoid causing discomfort. A. Hemoglobin level
b) Avoid taking a rectal temperature due to the presence of hemorrhoids. B. Total lymphocyte count
c) Inspect the anus before taking a rectal temperature. C. Serum albumin level
d) Use a different method to obtain the client's temperature. D. Urinary urea nitrogen level
99. A nurse is assessing the temperature of a client and notices that the client has 4. The nurse is assessing a patient's nutritional status and wants to evaluate the state
confusion and restlessness, but only a slight temperature elevation. The nurse of nitrogen balance. Which laboratory test would be most appropriate?
suspects there may be an underlying process. What should the nurse do next? A. Serum proteins
a) Administer antipyretic medication to reduce the temperature. B. Urinary urea nitrogen
b) Monitor the client's vital signs and observe for any further changes. C. Urinary creatinine
c) Discontinue temperature assessment as it may not be a valid indication of D. Total lymphocyte count
pathology. 5. The nurse is assessing a patient's nutritional status and wants to evaluate the total
d) Consult with the healthcare provider for further evaluation. number of lymphocyte white blood cells. Which laboratory test should the nurse
100. A nurse is assessing the temperature of an adult client and finds that the client consider?
has a fever. What is the most likely reason for the client's elevated body temperature? A. Hemoglobin level
a) Increased metabolic rate B. Serum albumin level
b) Decreased cardiac output C. Total lymphocyte count
c) Increased environmental temperature D. Urinary urea nitrogen level
d) Decreased efficiency of thermoregulation control processes 6. The nurse is caring for a client with a urinary catheter. Which intervention is essential
COMPREHENSIVE PHASE - RECALLS 1 (NP4) for maintaining catheter care?
1. Mrs. Johnson, a 65-year-old woman, visits the clinic for a routine check-up. During A. Emptying the drainage bag once a day – should be regularly
the assessment, the nurse asks Mrs. Johnson about her nutritional habits. Mrs. B. Disconnecting the catheter from the drainage tubing to irrigate it
Johnson mentions that she often skips meals and prefers eating processed foods. C. Securing the catheter to the client's leg with tape
Upon further inquiry, the nurse finds out that Mrs. Johnson has lost 15 pounds in the D. Cleaning the perineal area with soap and water daily
last 6 months. Which statement by Mrs. Johnson indicates high nutritional risk? 7. The nurse is assessing a client with urinary incontinence. Which intervention is
A. "I eat fewer than two meals per day." – can lead to nutrient deficiency and appropriate to promote continence?
weight loss A. Restricting fluid intake to minimize urine production
B. "I eat few fruits, vegetables, or milk products." B. Encouraging the client to hold urine for as long as possible
C. "I have an illness that made me change the kind of food I eat." C. Teaching pelvic floor muscle exercises (Kegel exercises)
D. "I take three or more different prescribed or over-the-counter drugs a day." D. Administering an antidiuretic medication to decrease urine output
2. A nurse is caring for a patient with a diagnosis of heart failure. The nurse is 8. The nurse is caring for a client with a urinary tract infection (UTI). Which intervention
monitoring the patient's fluid balance closely. Which of the following assessment is important for preventing the spread of infection?
findings would indicate fluid overload in a patient with heart failure? A. Encouraging the client to drink plenty of fluids – just flushing not prevent
A. Decreased blood pressure B. Administering antibiotics as ordered
B. Increased heart rate C. Providing perineal care after each voiding or bowel movement
C. Decreased urine output – ineffective pumping activity = decrease renal D. Applying heat to the suprapubic area to relieve discomfort
perfusion = decrease urinary output
9. The nurse is caring for a client with urinary retention. Which intervention is B. Granulation tissue fills in the wound bed.
appropriate to promote bladder emptying? C. The wound heals quickly with minimal scarring.
A. Encouraging the client to drink caffeinated beverages D. The wound heals from the inside out.
B. Assisting the client to a sitting position on the bedside commode. 16. A 35-year-old male patient presents to the clinic complaining of feeling dizzy and
C. Applying a cold pack to the lower abdomen – temporary relief lightheaded. Which vital sign should the nurse assess first?
D. Administering a diuretic medication to increase urine production A. Temperature
10. The nurse is caring for a client with a urinary diversion. Which intervention is B. Pulse
important for stoma care? C. Blood pressure – decreased BP = dizzy
A. Applying a sterile dressing over the stoma D. Respirations
B. Emptying the drainage bag when it is two-thirds full 17. A patient's blood pressure is 150/90 mmHg. How would you classify this blood
C. Irrigating the stoma with normal saline solution pressure reading?
D. Cleansing the stoma with mild soap and water A. Normal blood pressure
11. The nurse is caring for a client with a surgical wound. Which intervention is B. Prehypertension
essential for preventing surgical site infections? C. Hypertension stage 1
A. Administering prophylactic antibiotics after surgery D. Hypertension stage 2
B. Changing the wound dressing daily
C. Applying hydrogen peroxide to the wound
D. Using sterile gloves during wound care
12. The nurse is assessing a client's wound for signs of infection. Which finding is most
indicative of an infected wound?
A. Redness and swelling at the wound site
B. Clear serous drainage from the wound – normal finding
C. Presence of granulation tissue – (+) healing
D. Absence of pain at the wound site
13. The nurse is providing wound care for a client with a pressure ulcer. Which
18. A 2-year-old child is admitted to the pediatric unit. Which method should the nurse
intervention is appropriate for promoting wound healing?
use to assess the child's temperature?
A. Applying a dry sterile dressing to the wound
A. Oral
B. Irrigating the wound with hydrogen peroxide
B. Axillary
C. Using a transparent film dressing over the wound
C. Tympanic
D. Packing the wound with wet-to-dry dressings
D. Rectal
14. The nurse is providing wound care for a client with a surgical incision. Which
19. A patient's pulse rate is 110 beats per minute. How would you classify this pulse
intervention is important for preventing dehiscence?
rate?
A. Applying adhesive strips across the incision
A. Bradycardia
B. Leaving the incision exposed to air
B. Tachycardia
C. Using sterile gloves during wound care
C. Normal
D. Administering pain medication as prescribed
D. Irregular
15. The nurse is caring for a client with a wound that is healing by secondary intention.
20. A patient has a respiratory rate of 8 breaths per minute. How would you classify
Which statement accurately describes the healing process?
this respiratory rate?
A. The wound edges are brought together with sutures or staples.
A. Bradypnea
B. Tachypnea A. Blood pressure
C. Normal B. Respiratory rate
D. Apnea C. Pulse rate
21. A 70-year-old patient is admitted to the hospital with a diagnosis of pneumonia. D. Temperature
Which age-related change should the nurse consider when assessing the patient's C1 – C7 = Cervical Head, neck, diaphragm, deltoids, biceps,
vital signs? risk extenders, triceps
A. Increased core body temperature T1-T12 = Thoracic Chest and abdominal muscles
B. Decreased blood pressure L1-L5 = Lumbar Leg muscles
C. Decreased respiratory rate S1-S5 = Sacral Bowel, Bladder, sexual function
D. Increased pulse rate 26. A patient with a history of chronic obstructive pulmonary disease (COPD) is
22. A nurse is assessing a newborn infant's vital signs. Which vital sign should the admitted to the hospital. Which vital sign finding should the nurse prioritize in this
nurse expect to be significantly different from an adult's vital sign? patient?
A. Blood pressure A. Blood pressure
B. Respiratory rate B. Temperature
C. Temperature C. Pulse oximetry – WOF: hypoxemia and respiratory compromise
D. Pulse rate D. Respiratory rate
23. A patient with a body mass index (BMI) of 35 is admitted to the hospital. How 27. A patient presents to the emergency department with a suspected myocardial
should the nurse consider the patient's BMI when assessing blood pressure? infarction. Which vital sign finding should the nurse consider as an early indicator of
A. BMI does not affect blood pressure complications?
B. BMI may cause falsely elevated blood pressure readings – increased BMI = A. Blood pressure – drop of BP =cardiogenic shock = complication
increase adipose tissue = compressed blood vessels = increase BP (falsely) B. Temperature
C. BMI may cause falsely low blood pressure readings C. Heart rate
D. BMI may cause irregular blood pressure readings D. Respiratory rate
BMI Weight Status 28. A patient with end-stage renal disease is undergoing hemodialysis. Which vital sign
<18.5 Underweight should the nurse closely monitor during the dialysis session?
18.5-24.9 Normal A. Blood pressure
25-29.9 Overweight B. Temperature
30-39.9 Obese C. Heart rate
>40 Extremely Obese D. Respiratory rate
BMI =
𝑤𝑡 (𝑘𝑔) 29. A patient with a traumatic brain injury is admitted to the intensive care unit. Which
ℎ𝑡 (𝑚2 )
vital sign finding should the nurse be most concerned about in this patient?
24. A patient with a history of anxiety disorder is admitted to the emergency
A. Blood pressure
department. When assessing the patient's vital signs, which finding should the nurse
B. Temperature
anticipate?
C. Heart rate
A. Elevated body temperature
D. Intracranial pressure – increase ICP = serious neurological complication;
B. Bradycardia – should be tachy
priority
C. Tachypnea
D. Hypertension
25. A patient with a spinal cord injury at the C4 level is admitted to the rehabilitation
unit. Which vital sign should the nurse pay particular attention to in this patient?
"According to Republic Act No. 9173, what is the specific provision that guarantees the
delivery of quality basic health services through an adequate nursing personnel
system throughout the country?"
A. Section 2
B. Section 5
C. Section 7
E. Section 9
32. A nurse is attending a seminar on the regulatory aspects of nursing practice. The
speaker discusses the qualifications of the Chairperson and Members of the Board as
stated in Republic Act No. 9173. The nurse is asked: "According to Section 4 of
✓ ICP = Pressure in the head Republic Act No. 9173, what is the specific qualification required for the Chairperson
✓ MAP = Pressure in the body and Members of the Board regarding their education?"
✓ CPP = Amount of blood needed to supply adequate O2 to the brain A. Possession of a master's degree in nursing
30. A patient is receiving intravenous chemotherapy for cancer treatment. Which vital B. Completion of a doctoral degree in nursing
sign finding should the nurse closely monitor for potential complications? C. Completion of a bachelor's degree in nursing
A. Blood pressure D. Possession of a medical degree
B. Temperature 33. A nursing student is preparing for their future career as a nurse and wants to
C. Heart rate understand the role of the Board of Nursing as outlined in Republic Act No. 9173. They
D. Respiratory rate come across the following question: "According to Republic Act No. 9173, what
specific power does the Board of Nursing have in relation to nursing education?"
A. Conducting hearings and investigations for unethical conduct
B. Recognizing nursing specialty organizations
C. Ensuring quality nursing education by examining facilities
D. Conducting the licensure examination for nurses
34. A nurse is reviewing their professional responsibilities and obligations as outlined
in Republic Act No. 9173. They encounter the following question: "According to
Republic Act No. 9173, what is the specific provision regarding the Code of Ethics for
nurses?"
A. Section 6
B. Section 8
C. Section 10
D. Section 12
35. A nursing student is studying the requirements for admission to the licensure
examination as stated in Republic Act No. 9173. They read the following question:
"According to Republic Act No. 9173, what is the specific qualification required for
admission to the licensure examination?"
A. Completion of a master's degree in nursing
31. A nursing student is studying for their upcoming licensure examination and comes B. Good moral character
across a question related to Republic Act No. 9173. They read the following: C. Filipino citizenship
D. Completion of a doctoral degree in nursing D. Section 18
36. A nursing student is reviewing the provisions of Republic Act No. 9173 and Scenario: You are a nursing instructor conducting a class on professional ethics for
encounters the following question: "According to the Act, what is the specific a group of nursing students. You want to assess their understanding of the Code of
requirement for the Chairperson and Members of the Board of Nursing in terms of Ethics for Registered Nurses. You decide to create a set of questions based on the
continuous practice of the nursing profession?" provided PDF document.
A. At least five years 41. According to the Code of Ethics for Registered Nurses, what is the responsibility
B. At least seven years of a registered nurse when the preservation of health, prevention of illness, alleviation
C. At least ten years of suffering, and restoration of health are not possible?
D. At least twelve years A. Assisting towards a peaceful death
37. A nursing student is studying the role of the Board of Nursing as outlined in B. Promoting health and preventing illness
Republic Act No. 9173. They come across the following question: "According to the C. Preserving health at all costs
Act, what specific power does the Board of Nursing have in relation to the licensure D. Alleviating suffering and restoring health
examination for nurses?" 42. According to the Code of Ethics for Registered Nurses, what should Registered
A. Determining the passing score for the examination nurses do with personal information acquired in the process of giving nursing care?
B. Developing the content and format of the examination A. Share it with other healthcare professionals
C. Administering the examination to candidates B. Hold it in strict confidence
D. Evaluating the results and issuing the license C. Disclose it to the patient's family
38. A nurse is attending a professional development seminar on ethical conduct in D. Use it for research purposes
nursing practice. The speaker discusses the disciplinary actions outlined in Republic 43. According to the Code of Ethics for Registered Nurses, what should register nurses
Act No. 9173. The nurse is asked: "According to the Act, what specific action may lead consider when providing nursing care to patients?
to the removal or suspension of a nurse's license?" A. Individuality and totality of patients
A. Violation of patient confidentiality B. Cultural diversity and political status
B. Failure to maintain continuing education requirements C. Social and ecological aspects of illness
C. Conviction of a criminal offense involving moral turpitude D. Physiological and psychological aspects of illness
D. Non-compliance with nursing practice standards 44. According to the Code of Ethics for Registered Nurses, what should register nurses
39. A nursing student is reviewing the requirements for admission to the licensure do in terms of patients' rights?
examination as stated in Republic Act No. 9173. They read the following question: A. Uphold the rights of individuals
"According to the Act, what is the specific educational requirement for admission to the B. Respect the cultural and values of patients
licensure examination?" C. Provide a favorable environment for growth and development
A. Completion of a bachelor's degree in nursing D. Ensure patients' records are available only to those involved in their care
B. Completion of a master's degree in nursing 45. According to the Code of Ethics for Registered Nurses, what is the responsibility
C. Completion of a doctoral degree in nursing of registered nurses in terms of documentation?
D. Completion of a diploma program in nursing A. Accurate documentation is the hallmark of nursing accountability
40. A nurse is researching the provisions related to the Code of Ethics for nurses as B. Documentation should be shared with other healthcare providers
stated in Republic Act No. 9173. They encounter the following question: "According to C. Documentation should be used for research purposes
the Act, what is the specific provision regarding the Code of Ethics for nurses?" D. Documentation should be held in strict confidence
A. Section 6 46. According to the Code of Ethics for Registered Nurses, what is the role of
B. Section 10 registered nurses in relation to the growth and development of other registered nurses
C. Section 14 in their charge?
A. Providing mentorship and guidance B. Humoral immune response – driven by B-cells, develops quickly
B. Ensuring compliance with regulatory standards C. Phagocytic immune response – involves white blood cell that have the ability
C. Facilitating professional programs for specialty certification to ingest foreign particles
D. Creating a favorable work environment D. Complement immune response – series of enzymatic proteins in the serum
47. According to the Code of Ethics for Registered Nurses, what should register nurses that when activated, it destroys bacteria and other cells
do when faced with conflicts between the cultural values of patients and the provision 53. A nurse is teaching a patient about the function of memory cells in the immune
of nursing care? system. Which of the following statements accurately describes memory cells?
A. Prioritize the welfare and safety of patients A. Memory cells are responsible for recognizing antigens from previous exposure
B. Respect the cultural values of patients above all else and mounting an immune response.
C. Seek guidance from healthcare administrators B. Memory cells are lymphocytes that lyse cells infected with a virus. – cytotoxic
D. Consult with other healthcare professionals T-cells
48. According to the Code of Ethics for Registered Nurses, what should register nurses C. Memory cells decrease B-cell activity to a level compatible with life. –
do to ensure the quality of nursing care and practice? suppressor T-cells
A. Seek feedback from patients and families D. Memory cells are lymphocytes that defend against microorganisms and
B. Continuously update their knowledge and skills malignant cells. – Natural killer cells
C. Collaborate with interdisciplinary healthcare teams 54. A nurse is assessing a patient's immune function. Which of the following terms
D. Comply with regulatory standards and guidelines. refers to the coating of antigen-antibody molecules with a sticky substance to facilitate
49. According to the Code of Ethics for Registered Nurses, what should register nurses phagocytosis?
do to effectively render appropriate nursing services? A. Agglutination
A. Collaborate with other healthcare professionals B. Opsonization
B. Acquire and develop necessary competence C. Complement – series of enzymatic proteins in the serum that when activated,
C. Advocate for improved healthcare policies it destroys bacteria and other cells
D. Foster a supportive work environment D. Apoptosis
50. According to the Code of Ethics for Registered Nurses, what is the overarching 55. A patient with a primary immune deficiency is experiencing a gradual deterioration
principle that guides the ethical conduct of registered nurses? of the immune system due to the aging process. What term is used to describe this
A. Respect for individual autonomy phenomenon?
B. Commitment to social justice A. Agglutination
C. Promotion of health and well-being B. Immunosenescence
D. Preservation of human dignity C. Immunoregulation – complex system of checks and balances that regulates or
51. A nurse is assessing a patient's immune function. Which of the following terms control immune responses
refers to a substance that induces the production of antibodies? D. Interferons – protein formed when cells are exposed to a viral or foreign agents,
A. Agglutination -clumping effect that occurs when antibody cross-link between 2 capable of activating other components of the immune system
antigens 56. A nurse is caring for a patient with an immune system dysfunction. Which of the
B. Antibody – proteins developed by the body in response to antigens following terms refers to the study of diseases resulting from dysfunctions within the
C. Antigen – triggers the production of antibodies immune system?
D. Apoptosis – programmed cell death A. Immunopathology
52. A patient has been diagnosed with a condition that involves the attack of pathogens B. Immunoregulation
by T cells. Which immune response is this patient experiencing? C. Immunosenescence
A. Cellular immune response – driven by T-cells, takes longer time to develop D. Interferons
57. A nurse is assessing a patient's immune function. Which of the following cells are 62. A 32-year-old male patient presents with a localized skin rash that is red, itchy, and
important for producing a humoral immune response? has raised bumps. The patient reports a history of allergies and has a family history of
A. B cells atopic dermatitis. After examination, the healthcare provider diagnoses the patient with
B. Cytotoxic T cells – lymphocytes that lyse cells infected with virus atopic dermatitis. Which of the following immune mechanisms is primarily involved in
C. Helper T cells – lymphocytes that attack foreign invaders directly the development of atopic dermatitis?
D. Natural killer (NK) cells - lymphocytes that defends body against A. Type I hypersensitivity reaction – immediate reaction by IgE antibody
microorganism and malignant cells B. Type II hypersensitivity reaction – (Cytotoxic) antigen-antibody complex; BT
58. A nurse is teaching a patient about the process of genetic engineering. Which of mix match, antibody bind antigen cell = cell lysis = phagocytosis/inflammation
the following best describes genetic engineering? = tissue damage
A. A process that enables replacement of missing or defective genes. C. Type III hypersensitivity reaction – antigen-antibody complement (cascade);
B. A process that activates other components of the immune system – interferons series of inflammatory responses; SLE; RA;Glumerulonephritis
C. A process that regulates or controls immune responses – immunoregulation D. Type IV hypersensitivity reaction – delayed hypersensitivity; graft rejection;
D. A process that lyses cells infected with a virus – cytotoxic T-cells contact dermatitis
59. A nurse is assessing a patient's immune function. Which of the following terms 63. A 60-year-old male patient with a history of chronic alcohol abuse presents with
refers to the specific area of an antigen that binds with an antibody combining site and abdominal pain, jaundice, and ascites. Laboratory tests reveal decreased serum
determines the specificity of the antigen-antibody reaction? albumin levels and prolonged prothrombin time. The healthcare provider suspects liver
A. Agglutination cirrhosis. Which of the following immune mechanisms is involved in the pathogenesis
B. Antibody of liver cirrhosis?
C. Antigenic determinant A. Autoimmune reaction
D. Stem cells – the precursors of all blood cells B. Immune complex deposition
60. A nurse is caring for a patient who has undergone a bone marrow transplant. Which C. Chronic inflammation – sustained immune response = fibrosis and tissue
of the following cells are responsible for recognizing self-antigens and maintaining scarring
immune tolerance? D. Alloimmunity – body attacks the transplanted tissues = graft rejections
A. B cells – humoral 64. A 25-year-old female patient presents with joint pain, swelling, morning stiffness,
B. Natural killer (NK) cells – against malignant cells and fatigue. On physical examination, the healthcare provider observes swelling and
C. Suppressor T cells tenderness in multiple joints, including the metacarpophalangeal joints and wrists.
D. Memory cells – recognizing antigens from previous exposure Laboratory tests reveal elevated rheumatoid factor and anti-cyclic citrullinated peptide
61. A 45-year-old female patient presents with recurrent respiratory infections, chronic (anti-CCP) antibodies. Which of the following conditions is most likely causing the
diarrhea, and unexplained weight loss. She reports a history of autoimmune disorders patient's symptoms?
in her family. Upon further investigation, it is discovered that the patient has low levels A. Rheumatoid arthritis (RA)
of immunoglobulins. Which of the following conditions is most likely causing the B. Systemic lupus erythematosus (SLE) – autoimmune = cells mistakenly attack
patient's symptoms? healthy cells
A. Selective IgA deficiency – primary immunodeficiency; asymptomatic or mild C. Sjogren's syndrome – drying of 3 glands = salivary glands, lacrimal gland,
symptoms Bartholin gland
B. Common variable immunodeficiency (CVID) – low levels of immunoglobulins; D. Ankylosing spondylitis – inflammatory disease that can cause spine and joint
recurrent respiratory infections, during adulthood, associated with autoimmune stiffness
d/o 65. A neonate is born with recurrent bacterial and fungal infections, failure to thrive,
C. DiGeorge syndrome – infancy, recurrent severe infections and failure to thrive and chronic diarrhea. Laboratory tests reveal low levels of immunoglobulins and
D. Severe combined immunodeficiency (SCID)
absence of lymphoid tissue. Which of the following conditions is most likely causing
the neonate's symptoms?
A. Severe combined immunodeficiency (SCID)
B. DiGeorge syndrome – detect in t-cell development
C. Hyper IgM syndrome – primary immunodeficient which specific antibody
production defects
D. Wiskott-Aldrich syndrome
66. A 55-year-old male presents to the emergency department with sudden onset of
weakness on the right side of his body. He is unable to speak properly and has difficulty
understanding speech. On examination, his face appears droopy on the right side and
he has weakness in his right arm and leg. His vital signs are stable. What is the most
likely diagnosis?
A. Ischemic stroke
B. Subarachnoid hemorrhage – “worst headache of my life”
C. Migraine with aura – characterized by visual disturbances = flashing lights,
blind spots
69. A 45-year-old female presents to the clinic with complaints of severe headache,
D. Bell's palsy – sudden weakness or paralysis of one side of the pain
blurred vision, and vomiting. On examination, her blood pressure is elevated, and she
67. A 65-year-old female presents to the clinic with complaints of progressively
has papilledema on fundoscopic examination. What is the most likely diagnosis?
worsening memory loss, difficulty finding words, and getting lost in familiar places. She
A. Hypertensive crisis
also experiences changes in mood and personality. On examination, she has impaired
B. Migraine headache
judgment, poor insight, and difficulty with problem-solving tasks. What is the most likely
C. Acute angle-closure glaucoma
diagnosis?
D. Increased intracranial pressure
A. Alzheimer's disease
Papilledema – increase pressure on the brain causing optic nerve swelling
B. Parkinson's disease – a progressive disorder that is caused by degeneration
70. A 70-year-old male with a history of hypertension and diabetes mellitus presents
of nerve cells in the part of the brain called the substantia nigra, which controls
to the clinic with complaints of progressive weakness and numbness in his legs. On
movement
examination, he has decreased sensation and proprioception in his lower extremities,
C. Multiple sclerosis – a potentially disabling disease of the brain and spinal cord
absent deep tendon reflexes, and a positive Romberg test. What is the most likely
(central nervous system)
diagnosis?
D. Huntington's disease – an inherited disorder that causes nerve cells (neurons)
A. Diabetic neuropathy
in parts of the brain to gradually break down and die. “chorea”
B. Guillain-Barré syndrome – ascending weakness
68. A 30-year-old male is brought to the emergency department after a motorcycle
C. Peripheral arterial disease – narrowing or blockage of the arteries, supplying
accident. He is unresponsive with decerebrate posturing, dilated pupils, and absent
the extremities
corneal reflexes. His respiratory rate is slow and irregular. What is the most likely
D. Spinal cord compression
cause of his condition?
71. A 55-year-old female patient presents with recurrent upper respiratory infections,
A. Traumatic brain injury
chronic sinusitis, and bronchiectasis. Laboratory tests reveal elevated sweat chloride
B. Spinal cord injury – motor and sensory deficits
levels. The patient's medical history is significant for infertility. Which of the following
C. Ischemic stroke – neurological deficits
conditions is most likely causing the patient's symptoms?
D. Parkinson's disease – motor deficits
A. Cystic fibrosis (CF) – CFTR = defective gene = Na and Cl ions regulation
across membranes
B. Chronic granulomatous disease (CGD) C. Wiskott-Aldrich syndrome
C. Severe combined immunodeficiency (SCID) D. Transfusion-associated graft-versus-host disease (TA-GVHD).
D. Wiskott-Aldrich syndrome - characterized by abnormal immune system 76. A 50-year-old male patient presents with joint pain, fatigue, and dry eyes and
function (immune deficiency), eczema (an inflammatory skin disorder mouth. Laboratory tests reveal elevated antinuclear antibodies (ANA) and anti-
characterized by abnormal patches of red, irritated skin), and a reduced ability SSA/Ro antibodies. The patient's medical history is significant for chronic hepatitis C
to form blood clots. infection. Which of the following conditions is most likely causing the patient's
72. A 30-year-old male patient presents with recurrent bacterial skin infections, symptoms?
abscesses, and impaired wound healing. Laboratory tests reveal neutrophilia and a A. Systemic lupus erythematosus (SLE)
defective respiratory burst in phagocytic cells. The patient's medical history is B. Sjogren's syndrome.
significant for recurrent infections since childhood. Which of the following conditions is C. Rheumatoid arthritis (RA)
most likely causing the patient's symptoms? D. Mixed connective tissue disease (MCTD)
A. Chronic granulomatous disease (CGD). 77. A 30-year-old female patient presents with recurrent sinus and lung infections,
B. Bruton's agammaglobulinemia chronic diarrhea, and unexplained weight loss. Laboratory tests reveal low levels of
C. Hyper IgM syndrome immunoglobulins and absent lymphoid tissue. The patient's medical history is
D. DiGeorge syndrome significant for a history of radiation therapy for Hodgkin's lymphoma. Which of the
73. A 45-year-old female patient presents with joint pain, fever, and a rash on her following conditions is most likely causing the patient's symptoms?
cheeks and nose. Laboratory tests reveal elevated antinuclear antibodies (ANA) and A. Common variable immunodeficiency (CVID)
anti-double-stranded DNA (anti-dsDNA) antibodies. The patient's medical history is B. Severe combined immunodeficiency (SCID)
significant for photosensitivity and oral ulcers. Which of the following conditions is most C. Wiskott-Aldrich syndrome
likely causing the patient's symptoms? D. Radiation-induced immunodeficiency.
A. Systemic lupus erythematosus (SLE). 78. A 40-year-old male patient presents with recurrent skin infections,
B. Rheumatoid arthritis (RA) hepatosplenomegaly, and lymphadenopathy. Laboratory tests reveal leukocytosis with
C. Sjogren's syndrome abnormal lymphocyte morphology. The patient's medical history is significant for a
D. Ankylosing spondylitis family history of similar symptoms. Which of the following conditions is most likely
74. A 25-year-old female patient presents with recurrent bacterial and fungal skin causing the patient's symptoms?
infections. Laboratory tests reveal normal immunoglobulin levels and lymphocyte A. Chronic lymphocytic leukemia (CLL)
count. The patient's medical history is significant for a recent kidney transplant. Which B. Hairy cell leukemia (HCL). – lymphoproliferative disorder characterized by
of the following conditions is most likely causing the patient's symptoms? abnormal B-cells with hairlike projections in the bone marrow and spleen.
A. Immunosuppression due to transplant medications. C. Sezary syndrome
B. Hyper IgE syndrome (Job syndrome) D. Lymphomatoid granulomatosis
C. DiGeorge syndrome 79. A 65-year-old patient presents to the clinic with complaints of joint pain and
D. X-linked agammaglobulinemia (XLA) stiffness in the hands and knees. The patient reports difficulty in performing daily
75. A 35-year-old male patient presents with recurrent respiratory infections, chronic activities such as buttoning clothes and walking. On examination, the nurse notes
diarrhea, and failure to thrive since childhood. Laboratory tests reveal low levels of swelling and tenderness in multiple joints. Which of the following conditions is most
immunoglobulins and absent lymphoid tissue. The patient's medical history is likely causing the patient's symptoms?
significant for a history of multiple blood transfusions. Which of the following conditions A. Osteoporosis – decreased bone density
is most likely causing the patient's symptoms? B. Rheumatoid arthritis – autoimmune
A. Common variable immunodeficiency (CVID) C. Osteoarthritis – wear and tear of joints; affects weight bearing joints (hips,
B. Severe combined immunodeficiency (SCID) spine, knees, fingers)
D. Gout – uric acid crystals deposition 84. A 55-year-old male patient presents to the clinic with complaints of joint pain and
80. A 50-year-old male patient presents to the emergency department with severe stiffness in the hands, wrists, and knees. The patient reports morning stiffness that
back pain. The pain started suddenly and is described as sharp and stabbing. The lasts for less than an hour. On examination, the nurse notes swelling and tenderness
patient reports that the pain worsens with movement and is accompanied by in the affected joints, as well as Heberden's nodes in the distal interphalangeal joints.
numbness and weakness in the legs. On examination, the nurse notes decreased Laboratory tests reveal normal levels of rheumatoid factor. Based on the clinical
sensation and strength in the lower extremities. Which of the following conditions is presentation, which of the following conditions is the most likely cause of the patient's
the most likely cause of the patient's symptoms? symptoms?
A. Osteoarthritis A. Osteoarthritis
B. Rheumatoid arthritis OA PIP DIP
C. Herniated intervertebral disc Heberden’s nodes Bony swelling
D. Osteoporosis Bouchard’s nodes Bony Swelling
81. A 30-year-old female patient presents to the clinic with complaints of joint pain and B. Rheumatoid arthritis
swelling in the wrists, fingers, and knees. The patient reports morning stiffness that RA PIP DIP
lasts for more than an hour. On examination, the nurse notes tender, warm, and Boutonniere Flexed Hyperextend
swollen joints. Laboratory tests reveal the presence of rheumatoid factor and elevated Swans neck Hyper flexed Flexed
C-reactive protein. Which of the following is the most appropriate initial intervention for ✓ Ulnar drift/ulnar deviation
this patient? ✓ Rheumatoid nodules(chronic)
A. Prescribe nonsteroidal anti-inflammatory drugs (NSAIDs) – symptomatic relief C. Gout
B. Administer corticosteroid injections into affected joints – temporary relief D. Psoriatic arthritis
C. Refer the patient to physical therapy for exercises 85. A 35-year-old female patient presents to the emergency department with acute
D. Start disease-modifying antirheumatic drugs (DMARDs) – Drug of choice onset of severe joint pain, redness, and swelling in the big toe. The patient reports a
82. A 60-year-old male patient with a history of osteoporosis is admitted to the hospital history of similar episodes in the past. On examination, the nurse notes a hot, tender,
after sustaining a fragility fracture of the hip. The nurse is providing education to the and swollen joint. Laboratory tests reveal elevated serum uric acid levels. Which of the
patient about measures to prevent future fractures. Which of the following instructions following conditions is the most likely cause of the patient's symptoms?
should the nurse include in the teaching? A. Osteoarthritis
A. Perform weight-bearing exercises daily B. Rheumatoid arthritis
B. Limit calcium intake to prevent kidney stones C. Gout
C. Avoid exposure to sunlight to prevent skin cancer D. Psoriatic arthritis
D. Take bisphosphonate medication on an empty stomach 86. A 25-year-old male patient presents to the clinic with complaints of chronic back
83. A 45-year-old female patient presents to the clinic with complaints of joint pain, pain and stiffness that worsens with rest and improves with exercise. The patient
fatigue, and morning stiffness that lasts for more than an hour. The patient reports a reports a family history of similar symptoms. On examination, the nurse notes limited
family history of autoimmune diseases. On examination, the nurse notes tender, range of motion in the lumbar spine and tenderness in the sacroiliac joints. Laboratory
swollen joints and the presence of rheumatoid nodules. Laboratory tests reveal tests reveal the presence of HLA-B27 antigen. Which of the following conditions is the
elevated rheumatoid factor and anti-cyclic citrullinated peptide (anti-CCP) antibodies. most likely cause of the patient's symptoms?
Which of the following is the most likely diagnosis for this patient? A. Osteoarthritis
A. Systemic lupus erythematosus (SLE) B. Rheumatoid arthritis
B. Psoriatic arthritis C. Ankylosing spondylitis – HLA-B27 Antigen = (+) autoimmune disorders; detect
C. Ankylosing spondylitis ankylosing spondylitis and writer’s syndrome = reactive arthritis
D. Rheumatoid arthritis (RA). D. Psoriatic arthritis
87. A 65-year-old male presents to the clinic with complaints of joint pain and stiffness D. Sjögren's syndrome
in his hands and knees. He reports that the pain is worse in the morning and improves 91. A 60-year-old male presents with joint pain and stiffness in his knees, hips, and
throughout the day. On physical examination, there is tenderness and swelling in hands. He reports that the pain worsens with activity and improves with rest. On
multiple joints. Laboratory tests reveal an elevated erythrocyte sedimentation rate physical examination, there is bony enlargement and crepitus in the affected joints.
(ESR) and C-reactive protein (CRP) level. Which of the following conditions is the most Laboratory tests reveal an elevated erythrocyte sedimentation rate (ESR). Which of
likely diagnosis for this patient? the following conditions is the most likely diagnosis for this patient?
A. Osteoarthritis A. Osteoarthritis.
B. Rheumatoid arthritis. B. Rheumatoid arthritis
C. Gout C. Gout
D. Systemic lupus erythematosus D. Systemic lupus erythematosus
88. A 40-year-old female presents with aching and stiffness in her lower back that is 92. A 45-year-old male presents to the emergency department with severe chest pain
worse in the morning and improves with activity. She reports a history of alternating that radiates to his left arm. He is diaphoretic and appears anxious. His blood pressure
bouts of constipation and diarrhea. On physical examination, there is tenderness over is 160/100 mmHg, heart rate is 110 bpm, and ECG shows ST-segment elevation in
the sacroiliac joints. Which of the following conditions is the most likely diagnosis for leads II, III, and AVF. Which of the following conditions is the most likely diagnosis for
this patient? this patient?
A. Fibromyalgia – chronic disorder characterized by widespread musculoskeletal A. Myocardial infarction.
pain B. Pulmonary embolism – sudden onset DOB, pleurisy, right heart strain
B. Ankylosing spondylitis. C. Aortic dissection – sudden onset severe tearing or ripping chest pain that
C. Osteoporosis radiates to the back
D. Osteoarthritis D. Pericarditis – sharp pleuritic chest pain
89. A 55-year-old male presents with sudden severe pain and swelling in his right big 93. A 35-year-old female presents with sudden-onset dyspnea, pleuritic chest pain,
toe. He reports a history of similar episodes in the past. On physical examination, the and hemoptysis. She recently underwent surgery and has been immobile for the past
affected joint is warm, red, and tender to touch. Laboratory tests reveal an elevated few days. On physical examination, there are signs of right heart strain, including a
serum uric acid level. Which of the following conditions is the most likely diagnosis for tachycardia and hypoxemia. Which of the following conditions is the most likely
this patient? diagnosis for this patient?
A. Osteoarthritis A. Myocardial infarction
B. Rheumatoid arthritis B. Pulmonary embolism.
C. Gout C. Aortic dissection
D. Psoriatic arthritis D. Pericarditis
90. A 30-year-old female presents with joint pain and swelling in her hands, along with 94. A 60-year-old male presents with sudden-onset severe tearing or ripping chest
morning stiffness that lasts for more than an hour. She also reports fatigue and pain that radiates to the back. He also reports a difference in blood pressure between
occasional low-grade fever. On physical examination, there is synovitis in multiple his two arms. On physical examination, he has hypertension. Which of the following
joints, including the wrists and metacarpophalangeal joints. Laboratory tests reveal the conditions is the most likely diagnosis for this patient?
presence of rheumatoid factor and anti-cyclic citrullinated peptide (anti-CCP) A. Myocardial infarction
antibodies. Which of the following conditions is the most likely diagnosis for this B. Pulmonary embolism
patient? C. Aortic dissection.
A. Osteoarthritis D. Pericarditis
B. Rheumatoid arthritis. 95. A 25-year-old female presents with sharp, pleuritic chest pain that is worse with
C. Lupus erythematosus inspiration. She reports a recent upper respiratory infection. On physical examination,
there is a pericardial friction rub on auscultation. Which of the following conditions is 100. Scenario: A 32-year-old sexually active female presents with complaints of lower
the most likely diagnosis for this patient? abdominal pain, fever, and a foul-smelling vaginal discharge. She reports having
A. Myocardial infarction multiple sexual partners and inconsistent use of barrier methods. On examination,
B. Pulmonary embolism there is tenderness upon palpation of the cervix. What is the most likely diagnosis?
C. Aortic dissection A. Bacterial vaginosis
D. Pericarditis. B. Trichomoniasis
96. A 50-year-old male presents with severe chest pain that radiates to his left arm. C. Vulvovaginal candidiasis
He is diaphoretic and appears anxious. His blood pressure is 170/100 mmHg, heart D. Pelvic inflammatory disease.
rate is 120 bpm, and ECG shows diffuse ST-segment elevation. Which of the following COMPREHENSIVE PHASE - RECALLS 1 (NP5)
conditions is the most likely diagnosis for this patient? 1. A nurse is establishing a therapeutic relationship with a client diagnosed with
A. Myocardial infarction. depression. The nurse demonstrates empathy by:
B. Pulmonary embolism A. Sharing personal experiences of overcoming depression.
C. Aortic dissection B. Showing concern and compassion for the client's feelings.
D. Pericarditis C. Providing solutions and advice for managing depressive symptoms.
97. A 55-year-old male presents with sudden-onset severe headache, dizziness, and D. Encouraging the client to focus on positive aspects of life.
blurred vision. On physical examination, his blood pressure is 220/120 mmHg, and 2. A nursing student is learning about the components of a therapeutic relationship.
there is a bruit heard over his carotid arteries. Which of the following conditions is the Which of the following factors promotes communication and enhances relationships in
most likely diagnosis for this patient? all aspects of the nurse's life?
A. Migraine A. Trust.
B. Subarachnoid hemorrhage B. Empathy
C. Temporal arteritis C. Genuine interest
D. Hypertensive emergency. D. Self-awareness
98. A 30-year-old female presents with a sudden-onset severe headache, nausea, and 3. A nurse is assessing a client's level of trust in the therapeutic relationship. Which of
vomiting. She reports that the headache is the worst she has ever experienced. On the following behaviors by the nurse can help build trust?
physical examination, there are signs of meningeal irritation, including neck stiffness A. Making verbal commitments and not following through on them.
and a positive Kernig sign. Which of the following conditions is the most likely B. Exhibiting inconsistent verbal and nonverbal communication.
diagnosis for this patient? C. Demonstrating consistency, honesty, and keeping promises.
A. Migraine D. Avoiding active listening and dismissing the client's concerns.
B. Subarachnoid hemorrhage. 4. A nursing student is learning about the importance of self-awareness in the nurse-
C. Temporal arteritis client relationship. Which of the following statements is true regarding self-awareness?
D. Hypertensive emergency A. Self-awareness is not necessary for effective therapeutic communication.
99. Scenario: A 28-year-old female presents to the clinic with complaints of itching B. Self-awareness allows nurses to recognize their own biases and prejudices.
and burning in her vaginal area. Upon examination, you notice a thick, white, cottage C. Self-awareness is only important in psychiatric nursing practice.
cheese-like discharge. The patient reports that she has had similar symptoms in the D. Self-awareness is solely focused on understanding the client's experiences.
past. What is the most likely diagnosis? 5. A nurse is working with a client who is resistant to receiving care. Which therapeutic
A. Bacterial vaginosis – thin, grayish white discharge and fishy odor role of the nurse would be most beneficial in this situation?
B. Trichomoniasis – yellow-green frothy discharge, vaginal itching and discomfort A. Teacher -
C. Vulvovaginal candidiasis. B. Caregiver – physical comfort and care
D. Pelvic inflammatory disease – lower abdominal pai, fever, vaginal discharge C. Advocate.
D. Parent surrogate – caring for clients that have developmental/intellectual A. Trust
disabilities B. Genuine Interest
6. A nurse is working with a client who is expressing suicidal thoughts. What is the C. Empathy.
appropriate next step in managing this situation? D. Acceptance
A. Encouraging the client to talk openly about their feelings. 12. A nurse is working with a client who has schizophrenia. The nurse consistently
B. Assuring the client that everything will be fine and they have support. follows through on commitments made to the client. Which behavior is the nurse
C. Contacting the client's family members to inform them of the situation. exhibiting to enhance trust in the nurse-client relationship?
D. Initiating a suicide risk assessment and implementing appropriate safety A. Caring
measures. B. Consistency.
7. A nurse is caring for a client with a history of substance abuse who is at risk for C. Understanding
relapse. Which nursing intervention is most appropriate in promoting relapse D. Respect
prevention? 13. A nurse is caring for a client with anxiety disorder. The nurse listens actively and
A. Encouraging the client to attend support group meetings. attentively to the client's concerns and experiences. Which therapeutic communication
B. Allowing the client to have occasional controlled substance use. technique is the nurse using?
C. Minimizing discussions about the client's substance abuse history. A. Reflection – nurse restates client’s words
D. Providing the client with unlimited access to prescription medications. B. Restatement – nurse repeats the main idea expressed by the client
8. A nurse is caring for a client diagnosed with schizophrenia who is experiencing C. Clarification – nurse asks for further information
auditory hallucinations. Which nursing intervention is most appropriate in managing D. Active listening.
the client's hallucinations? 14. A nurse is working with a client who has a history of substance abuse. The nurse
A. Encouraging the client to ignore the hallucinations. avoids making judgments about the client's behavior and accepts the client as a worthy
B. Engaging the client in reality-based activities. individual. Which component of the nurse-client relationship is the nurse
C. Providing constant reassurance that the hallucinations are not real. demonstrating?
D. Administering antipsychotic medication as prescribed. A. Trust
9. A nurse is providing care for a client with a history of depression who is experiencing B. Genuine Interest
suicidal ideation. Which nursing intervention is most appropriate in managing the C. Empathy
client's suicidal ideation? D. Acceptance.
A. Encouraging the client to isolate themselves from others. 15. A nurse is working with a client who has bipolar disorder. The nurse consistently
B. Monitoring the client closely for any signs of worsening symptoms. exhibits openness and objectivity in their interactions with the client. Which behavior
C. Minimizing discussions about the client's feelings and emotions. is the nurse demonstrating in the nurse-client relationship?
D. Collaborating with the client to develop a safety plan. A. Caring
10. A nurse is caring for a client with a diagnosis of hypertension. Which lifestyle B. Openness.
modification is most effective in managing hypertension? C. Objectivity
A. Reducing sodium intake. D. Respect
B. Increasing caffeine consumption. 16. A nurse is working with a client who has a history of self-harm. The nurse
C. Decreasing physical activity. demonstrates respect for the client's autonomy and involves them in decision-making
D. Consuming high-fat foods. regarding their treatment plan. Which ethical principle is the nurse upholding?
11. A nurse is establishing a therapeutic relationship with a client diagnosed with A. Autonomy.
depression. The nurse demonstrates genuine interest and empathy towards the client. B. Beneficence – do good
Which component of the nurse-client relationship is the nurse exhibiting? C. non-maleficence – do no harm
D. Justice – fairness to equal treatment 22. A nurse is working with a client who has an eating disorder. The nurse encourages
17. A nurse is caring for a client who has schizophrenia. The nurse demonstrates the client to express their feelings and emotions through art therapy. Which therapeutic
congruence by ensuring that their verbal and nonverbal communication align. Which technique is the nurse utilizing?
behavior is the nurse exhibiting? A. Self-disclosure
A. Openness B. Active listening
B. Objectivity C. Therapeutic use of self
C. Congruence D. Expressive therapy – use of creative outlets, such as art, music
D. Approachability 23. A nurse is caring for a client who is experiencing hallucinations. The nurse
18. A nurse is working with a client who has borderline personality disorder. The nurse maintains a calm demeanor and provides reassurance to the client. Which behavior is
sets clear boundaries and consistently enforces them. Which therapeutic technique is the nurse exhibiting?
the nurse using? A. Objectivity – they are able to view and respond to the client’s experiences
A. Self-disclosure – nurse share personal experiences to establish rapport without having personal bias or judgement
B. Limit-setting B. Boundaries – setting limits in the nurse-client relationship
C. Reframing – helping the client to see the situation in a different perspective C. Therapeutic use of self – intentionally using their personality, insights and
D. Suggestion judgements
19. A nurse is caring for a client who is experiencing severe anxiety. The nurse D. Nonjudgmental attitude
provides information about relaxation techniques and encourages the client to practice 24. A nurse is working with a client who is experiencing severe depression. The nurse
them. Which role of the nurse is being demonstrated in this scenario? provides emotional support and encourages the client to express their feelings. Which
A. Teacher. therapeutic technique is the nurse utilizing?
B. Caregiver A. Reflection
C. Advocate – stand up for client rights B. Encouragement
D. Parent surrogate C. Clarification
20. A nurse is working with a client who has a history of trauma. The nurse actively D. Self-disclosure
listens to the client's experiences and validates their emotions. Which therapeutic 25. A nurse is caring for a client who has a history of self-harm. The nurse maintains
communication technique is the nurse using? professional boundaries and avoids engaging in personal relationships with the client.
A. Silence Which aspect of the nurse-client relationship is the nurse demonstrating?
B. Reflection A. Confidentiality
C. Validation. B. Trustworthiness
D. Summarization – recapping the main points C. Professionalism.
21. A nurse is caring for a client who is experiencing delusions. The nurse maintains a D. Empathy
nonjudgmental attitude and avoids arguing with the client about their beliefs. Which 26. A client with a history of suicidal ideation is admitted to the psychiatric unit. The
behavior is the nurse exhibiting in the nurse-client relationship? client has been exhibiting aggressive behavior and has made several threats towards
A. Respect others. The healthcare team decides to implement seclusion as a safety measure.
B. Empathy – nurse’s ability to understand and share the feelings of the client What is seclusion?
C. Acceptance. A. The use of physical force to restrict a person's freedom of movement. - restraint
D. Trust – involves the client having confidence in the nurse’s presence, integrity, B. The involuntary confinement of a person in a locked room for direct visual
and reliability monitoring.
C. The restriction of a person's visitors and mail for safety purposes. – restriction
to the client
D. The requirement for clients to continue participating in treatment on an A. Psychiatric clients have the right to refuse treatment and visitors.
involuntary basis after discharge. – mandatory outpatient treatment B. Psychiatric clients have the right to leave the hospital at any time.
27. A client with a history of substance abuse has been admitted to a psychiatric facility C. Psychiatric clients have the right to unrestricted communication, including
due to dangerous behavior when under the influence. The client's treatment team phone calls.
decides that the client requires involuntary hospitalization. What is involuntary D. Psychiatric clients have the right to access their medical records upon request.
hospitalization? 32. A client with a history of schizophrenia is admitted to a psychiatric unit. The client's
A. The requirement for clients to continue participating in treatment on an family expresses concern about the client's safety and asks the nurse about the use
involuntary basis after discharge. of seclusion and restraint. What is the nurse's best response?
B. The use of physical force to restrict a person's freedom of movement. A. "Seclusion and restraint are commonly used interventions to manage
C. The involuntary confinement of a person in a locked room for direct visual aggressive behavior in psychiatric settings."
monitoring. B. "Seclusion and restraint are never used in psychiatric settings due to ethical
D. The commitment of a client to a facility for psychiatric care against their will considerations.".
until they no longer pose a danger to themselves or others. C. "Seclusion and restraint are only used as a last resort when other interventions
28. A client with severe and persistent mental illness has been court-ordered to have failed."
participate in outpatient treatment after being discharged from the hospital. This type D. "Seclusion and restraint are primarily used to punish clients for disruptive
of treatment is known as: behavior."
A. Seclusion 33. A client with a history of depression is admitted to a psychiatric facility. The client's
B. Restraint family expresses concern about the standard of care provided to their loved one. What
C. Voluntary hospitalization does the term "standard of care" refer to in psychiatric nursing?
D. Mandatory outpatient treatment. A. The level of care provided by healthcare professionals based on their personal
29. A client with a severe mental illness has been found to be gravely disabled and preferences.
unable to provide food, clothing, and shelter for themselves. The court appoints a legal B. The expectations of care based on cultural and religious beliefs.
guardian who assumes responsibilities for the client's best interests. This is known as: C. The legal requirements for providing care in psychiatric settings.
A. Involuntary hospitalization D. The expected level of care provided by healthcare professionals in a specific
B. Seclusion situation.
C. Conservatorship. 34. A client with schizophrenia is involved in a legal case where they are accused of
D. Restraint causing harm to another person during a psychotic episode. The client's defense
30. A client with a history of mental illness is being treated in an outpatient setting lawyer argues that the client should not be held responsible for their actions due to
rather than being hospitalized. This approach is based on the principle of: their mental illness. Which of the following types of torts is relevant to this situation?
A. Least restrictive environment. - A. Negligence. – failure to exercise reasonable care resulting in harm or injury to
B. Voluntary hospitalization – willingly seeking treatment and agreeing to be another person
hospitalized B. Invasion of privacy
C. Mandatory outpatient treatment C. Assault and battery
D. Involuntary hospitalization – commitment of a client to a facility for seeking care D. Intentional infliction of emotional distress
against their will.
31. A client with a history of bipolar disorder is admitted to a psychiatric hospital. The
client expresses concerns about their rights and asks the nurse about their rights as a
psychiatric patient. Which of the following statements accurately describes the rights
of psychiatric clients?
C. Administering antipsychotic medication as prescribed.
D. Engaging the client in reality-testing exercises.
39. A nurse is assessing a client with a history of alcohol use disorder who presents
with symptoms of withdrawal. Which of the following findings would the nurse expect
to observe in this client?
A. Elevated blood pressure and heart rate – autonomic hyperactivity
B. Increased appetite and weight gain.
C. Excessive sleepiness and sedation.
D. Elevated mood and increased energy.
35. A nurse is providing care to a client with a history of substance abuse who has 40. A nurse is providing education to a client with generalized anxiety disorder (GAD)
been court-ordered to participate in outpatient treatment after being discharged from who is prescribed medication therapy. The client asks the nurse about the expected
the hospital. The nurse explains to the client the concept of mandatory outpatient timeframe for the medication to take effect. What is the nurse's best response?
treatment. What does mandatory outpatient treatment entail? A. "The medication will start working immediately after the first dose."
A. The requirement for clients to continue participating in treatment on an B. "It may take a few weeks for the medication to reach its full effectiveness."
involuntary basis after discharge. C. "You will need to take the medication for a few months before noticing any
B. The use of physical force to restrict a person's freedom of movement during changes."
treatment. D. "The medication will start working within 24 hours of taking it."
C. The commitment of a client to a facility for psychiatric care against their will. 41. Mary, a 42-year-old woman, recently lost her husband in a car accident. She is
D. The requirement for clients to participate in treatment voluntarily and without experiencing intense sadness and difficulty accepting her loss. She constantly thinks
legal involvement. about her husband and has trouble sleeping. She isolates herself from friends and
36. A nurse is conducting a mental health assessment on a client who presents with family and has lost interest in activities she used to enjoy. Which term best describes
symptoms of depression. Which of the following statements made by the client would Mary's current condition?
indicate a potential risk for self-harm? A. Grief – subjective emotion and affect that are normal response to loss (sad,
A. "I've been feeling sad and unmotivated lately." anger, disbelief, DABDA)
B. "I don't enjoy doing the things I used to love." B. Mourning – outward expression of grief (attending funerals, wearing black,
C. "Sometimes I think about ending it all." sharing memories and other cultural/religious practices)
D. "I've been having trouble sleeping." C. Denial – refusing to acknowledge the reality
37. A nurse is caring for a client diagnosed with bipolar disorder who is currently D. Depression.
experiencing a manic episode. Which of the following interventions should the nurse 42. John, a 28-year-old man, recently lost his job due to company downsizing. He is
prioritize? experiencing feelings of worthlessness, hopelessness, and guilt. He has lost interest
A. Ensuring a quiet and calm environment for the client. in activities he used to enjoy and has trouble concentrating. He has thoughts of suicide
B. Encouraging the client to engage in physical activity. but denies any plans or intent to act on them. Which term best describes John's current
C. Administering a sedative medication to promote relaxation. condition?
D. Providing opportunities for social interaction with peers. A. Grief
38. A nurse is caring for a client diagnosed with schizophrenia who is experiencing B. Bereavement – process of experiencing the grief
auditory hallucinations. Which of the following interventions would be most appropriate C. Adjustment disorder – maladaptive reaction to a stressful life event
for managing the hallucinations? D. Major depressive disorder
A. Encouraging the client to interact with the hallucinations. 43. Sarah, a 65-year-old woman, recently lost her husband after a long battle with
B. Providing a quiet and calm environment for the client. cancer. She is experiencing intense sadness and has difficulty accepting her
husband's death. She frequently visits his grave and talks to him as if he is still alive. C. Mourning
She states that she can still feel his presence and believes he is watching over her. D. Anticipatory grief
Which term best describes Sarah's current condition? 48. Emma, a 60-year-old woman, recently lost her husband in a car accident. She
A. Complicated grieving – prolonged or intense grief response that already experiences intense sadness, guilt, and anger. She constantly thinks about her
impairs normal functioning husband and has difficulty concentrating. She has lost interest in activities she used
B. Disenfranchised grief – grief is not openly acknowledged or socially supported to enjoy and has trouble sleeping. Which term best describes Emma's current
C. Anticipatory grieving – manual thing that occur to a loss condition?
D. Adaptive denial – healthy A. Depression
44. Mark, a 35-year-old man, recently lost his job and is struggling to find new B. Bereavement
employment. He constantly worries about his financial situation, feels restless and on C. Complicated grieving.
edge, and has trouble sleeping. He experiences physical symptoms such as muscle D. Anticipatory grieving
tension and headaches. Which term best describes Mark's current condition? 49. Tom, a 28-year-old man, recently lost his job and is struggling with financial
A. Grief difficulties. He constantly worries about his future, experiences restlessness, and has
B. Adjustment disorder trouble sleeping. He feels fatigued and has difficulty concentrating. Which term best
C. Anxiety disorder. describes Tom's current condition?
D. Mourning A. Anxiety disorder.
45. Rachel, a 50-year-old woman, recently lost her mother to a terminal illness. She B. Depression
experiences intense sadness and frequently cries. She has difficulty sleeping and has C. Adjustment disorder
lost interest in activities she used to enjoy. She isolates herself from friends and family D. Bereavement
and has thoughts of joining her mother in death. Which term best describes Rachel's 50. Linda, a 50-year-old woman, recently lost her father after a long illness. She
current condition? experiences intense sadness, tearfulness, and difficulty accepting her loss. She
A. Grief frequently visits her father's grave and talks to him. She feels his presence and
B. Bereavement believes he is watching over her. Which term best describes Linda's current condition?
C. Complicated grieving. A. Bereavement
D. Adjustment disorder B. Complicated grief.
46. Sarah, a 32-year-old woman, recently lost her job and has been experiencing C. Disenfranchised grief
feelings of sadness, lack of motivation, and difficulty sleeping. She isolates herself D. Anticipatory grief
from friends and family and has lost interest in activities she used to enjoy. Which term 51. Emily, a 25-year-old woman, has been experiencing persistent feelings of sadness,
best describes Sarah's current condition? loss of interest in activities, changes in appetite, and difficulty concentrating. She has
A. Grief also been having thoughts of self-harm. Which condition is Emily most likely
B. Depression. experiencing?
C. Adjustment disorder A. Bipolar disorder
D. Bereavement B. Major depressive disorder.
47. Josh, a 45-year-old man, recently lost his wife to a terminal illness. He is C. Generalized anxiety disorder
experiencing intense sadness, persistent thoughts of his wife, and difficulty accepting D. Schizophrenia
her death. He avoids places and activities that remind him of her and has trouble 52. Jacob, a 40-year-old man, has been experiencing fluctuations in his mood. He has
sleeping. Which term best describes Josh's current condition? periods of elevated mood, increased energy, and decreased need for sleep. He
A. Grief engages in impulsive and risky behaviors during these episodes. Which condition is
B. Complicated grief. Jacob most likely experiencing?
A. Bipolar disorder. A. Schizophrenia.
B. Major depressive disorder B. Bipolar disorder
C. Generalized anxiety disorder C. Major depressive disorder
D. Schizophrenia D. Generalized anxiety disorder
57. John, a 45-year-old man, has a history of multiple episodes of depression
characterized by low mood, loss of interest, and feelings of worthlessness. He also
experiences periods of increased energy, racing thoughts, and impulsive behavior.
What is the most likely diagnosis for John?
A. Schizophrenia
B. Bipolar disorder.
C. Major depressive disorder
D. Generalized anxiety disorder
58. Emma, a 28-year-old woman, has been experiencing persistent worry and anxiety
53. Sarah, a 30-year-old woman, has been experiencing excessive worry and anxiety
about various aspects of her life, including work, relationships, and health. She often
about various aspects of her life. She often feels restless, fatigued, and has difficulty
feels restless, fatigued, and has difficulty concentrating. What is the most likely
concentrating. Which condition is Sarah most likely experiencing?
diagnosis for Emma?
A. Bipolar disorder
A. Schizophrenia
B. Major depressive disorder
B. Bipolar disorder
C. Generalized anxiety disorder.
C. Major depressive disorder
D. Schizophrenia
D. Generalized anxiety disorder.
54. Mark, a 35-year-old man, has been experiencing auditory hallucinations,
59. Josh, a 32-year-old man, has recently lost his job and has been feeling persistently
delusions, and disorganized thinking. He believes that he is being monitored by the
sad, hopeless, and has lost interest in activities he used to enjoy. He also experiences
government and that aliens are controlling his thoughts. Which condition is Mark most
changes in appetite and sleep patterns. What is the most likely diagnosis for Josh?
likely experiencing?
A. Schizophrenia
A. Bipolar disorder
B. Bipolar disorder
B. Major depressive disorder
C. Major depressive disorder.
C. Generalized anxiety disorder
D. Generalized anxiety disorder
D. Schizophrenia.
60. Sophie, a 25-year-old woman, has been experiencing intrusive thoughts and
55. Rachel, a 28-year-old woman, has been experiencing a persistent feeling of
repetitive behaviors such as handwashing and checking locks. She feels distressed
sadness, loss of interest in activities, changes in appetite, and difficulty sleeping. She
and spends a significant amount of time engaging in these rituals. What is the most
has been having thoughts of death and has lost a significant amount of weight recently.
likely diagnosis for Sophie?
Which condition is Rachel most likely experiencing?
A. Schizophrenia
A. Bipolar disorder
B. Bipolar disorder
B. Major depressive disorder.
C. Major depressive disorder
C. Generalized anxiety disorder
D. Obsessive-compulsive disorder.
D. Schizophrenia
61. Emily, a 28-year-old woman, has been recently diagnosed with major depressive
56. Sarah, a 35-year-old woman, has been admitted to the psychiatric unit with
disorder. She has been prescribed an antidepressant medication. Which of the
symptoms of paranoia, delusions, and disorganized behavior. She believes that her
following medications is commonly used as a first-line treatment for major depressive
neighbors are plotting against her and that she is being followed by government
disorder?
agents. What is the most likely diagnosis for Sarah?
A. Fluoxetine – SSRI; first-line treatment; work by increasing the availability of 66. A 35-year-old woman recently lost her husband in a car accident. She is
serotonin in the brain which helps improve mood experiencing intense sadness and has difficulty accepting the reality of his death. She
B. Alprazolam – benzodiazepines = anxiety disorders often finds herself talking to him as if he is still alive. Which stage of grieving, according
C. Lithium – mood stabilizer = bipolar disorder to Elisabeth Kübler-Ross, is this woman most likely experiencing?
D. Risperidone – atypical anti-psychotic medication = schizophrenia and bipolar A. Denial.
disorder with psychotic features B. Anger
62. Mark, a 40-year-old man, has been diagnosed with generalized anxiety disorder. C. Bargaining
He is experiencing persistent worry, restlessness, and muscle tension. Which class of D. Depression
medications is most commonly used as a first-line treatment for generalized anxiety Elisabeth Kübler-Ross
disorder? ✓ Denial
A. Selective serotonin reuptake inhibitors (SSRIs) - ✓ Anger
B. Benzodiazepines – short term used = risk for dependence and potential for ✓ Bargaining
sedation ✓ Depression
C. Mood stabilizers ✓ Acceptance
D. Antipsychotics 67. A 52-year-old man recently lost his job and is struggling to find new employment.
63. Sarah, a 25-year-old woman, is experiencing symptoms of a panic attack, including He feels worthless and doubts his abilities. He isolates himself from friends and family
palpitations, chest pain, and a fear of dying. Which class of medications is commonly and has lost interest in activities he once enjoyed. Which stage of grieving, according
used for the acute management of panic attacks? to George Engel, is this man most likely experiencing?
A. Benzodiazepines. A. Shock and disbelief
B. Selective serotonin reuptake inhibitors (SSRIs) – long term = panic disorders B. Developing awareness
C. Mood stabilizers C. Restitution
D. Antipsychotics D. Resolution of the loss.
64. David, a 30-year-old man, has been diagnosed with attention-deficit/hyperactivity George Engel
disorder (ADHD). He is experiencing symptoms of inattention, impulsivity, and ✓ Shock and disbelief
hyperactivity. Which class of medications is commonly used as a first-line treatment ✓ Developing awareness
for ADHD? ✓ Restitution
A. Stimulants – helps inattention, reduce hyperactivity, and control impulsivity; ✓ Resolving the loss
works by increasing the levels of dopamine and norepinephrine in the brain ✓ Idealization
B. Antidepressants – second line of treatment ✓ Outcome
C. Mood stabilizers. 68. A 28-year-old woman is diagnosed with terminal cancer. She becomes angry and
D. Antipsychotics resentful towards her religious beliefs, feeling that they have failed her. According to
65. Emma, a 50-year-old woman, has been diagnosed with schizophrenia and is Elisabeth Kübler-Ross, which stage of grieving is this woman most likely
experiencing positive symptoms such as hallucinations and delusions. Which class of experiencing?
medications is commonly used as a first-line treatment for schizophrenia? A. Denial
A. Antipsychotics. B. Anger.
B. Benzodiazepines C. Bargaining
C. Mood stabilizers D. Depression
D. Antidepressants 69. A 42-year-old man lost his wife in a car accident. He constantly questions what he
could have done differently to prevent the accident. He often says, "If only I had taken
a different route that day." According to Elisabeth Kübler-Ross, which stage of grieving A. Individual psychotherapy
is this man most likely experiencing? B. Group psychotherapy.
A. Denial C. Family therapy – includes the family members
B. Anger D. Cognitive therapy
C. Bargaining. 74. A 55-year-old male client is admitted to the psychiatric unit with symptoms of
D. Depression anxiety and panic attacks. The nurse implements a treatment approach that focuses
70. A 60-year-old woman recently lost her mother after a long battle with a terminal on gradually exposing the client to anxiety-provoking situations while providing
illness. She feels a deep sense of sadness and often finds herself crying relaxation techniques. Which psychosocial treatment modality is the nurse
uncontrollably. According to Elisabeth Kübler-Ross, which stage of grieving is this implementing?
woman most likely experiencing? A. Individual psychotherapy
A. Denial B. Group psychotherapy
B. Anger C. Systematic desensitization – dahan-dahan; flooding = biglaan
C. Bargaining D. Behavioral therapy
D. Depression. 75. A 40-year-old female client with schizophrenia is participating in a therapeutic
71. A 45-year-old female client comes to the clinic complaining of frequent nightmares community program. The program emphasizes the importance of social interactions,
and difficulty sleeping. She reports feeling anxious and on edge most of the time. Upon peer support, and a structured environment. Which psychosocial treatment modality
further assessment, the nurse discovers that the client experienced a traumatic event is the client participating in?
a few months ago. Which psychosocial theory explains the client's symptoms? A. Individual psychotherapy
A. Psychoanalytic theory – focuses on the unconscious mind and childhood B. Group psychotherapy
experiences; Sigmund Freud C. Therapeutic community.
B. Developmental theory – explains the behaviors and mental development; Erik D. Cognitive therapy
Eriksson 76. A 25-year-old female client is constantly making excuses for her failures and
C. Interpersonal theory. refuses to take responsibility for her actions. Which defense mechanism is the client
D. Behavioral theory employing?
72. 30-year-old male client is admitted to the psychiatric unit with symptoms of A. Denial
depression. He reports feeling hopeless, having difficulty concentrating, and losing B. Rationalization – unjustifiable excuses
interest in activities he used to enjoy. The nurse incorporates a treatment approach C. Projection – blaming others; blaming self = introjection
that focuses on the client's self-actualization and personal growth. Which psychosocial D. Regression – going back to early stage of development
treatment modality is the nurse implementing? 77. A 40-year-old male client consistently criticizes his colleagues for their mistakes,
A. Individual psychotherapy – one-on-one sessions between the client and yet fails to recognize his own errors. Which defense mechanism is the client
therapists employing?
B. Group psychotherapy – conducted in a group setting A. Displacement -
C. Cognitive therapy – focused on identifying and challenging the negative B. Repression – unconsciously forgetting
thought patterns Repression Suppression
D. Humanistic therapy. ✓ unconsciously forgetting ✓ consciously forgetting
73. A 20-year-old female client is participating in a support group for individuals with C. Projection.
eating disorders. The group provides a safe and non-judgmental environment for D. Intellectualization -
members to share their experiences and receive emotional support. Which
psychosocial treatment modality is the client participating in?
78. A 60-year-old female client has recently experienced the loss of her spouse. Rather current financial struggles and avoids seeking assistance. Which defense mechanism
than expressing her grief, she focuses on organizing the funeral arrangements and is the client employing?
taking care of practical matters. Which defense mechanism is the client employing? A. Denial.
A. Regression B. Projection
B. Repression C. Compensation
C. Sublimation – from being bad to good D. Idealization.
D. Suppression. 84. A 45-year-old female client, who is experiencing relationship difficulties, finds
79. A 35-year-old male client, who is an alcoholic, frequently blames his job-related solace in excessive shopping and accumulating unnecessary items. She believes that
stress for his drinking problem. He believes that if he didn't have a stressful job, he material possessions will fill the void in her life. Which defense mechanism is the client
wouldn't need to drink excessively. Which defense mechanism is the client employing? employing?
A. Rationalization. A. Reaction formation – action is incongruent to emotions
B. Undoing - a person tries to cancel out or remove an unhealthy, destructive or B. Repression
otherwise threatening thought or action by engaging in contrary behavior. C. Avoidance
C. Intellectualization D. Materialism.
D. Sublimation 85. A 50-year-old male client, who has a fear of public speaking, volunteers to give a
80. A 50-year-old female client experiences a traumatic event and has no recollection presentation at work to prove his competence. He believes that by facing his fear head-
of the event or any related memories. Which defense mechanism is the client on, he can overcome it. Which defense mechanism is the client employing?
employing? A. Intellectualization
A. Repression. B. Repression
B. Denial C. Displacement
C. Dissociation – an individual disconnects from their thoughts, memories, or D. Overcompensation.
identity to avoid stress 86. A patient with a diagnosis of schizophrenia is prescribed an antipsychotic
D. Regression medication. The nurse explains to the patient that the medication works by blocking
81. A 30-year-old male client has a history of childhood trauma. Despite facing certain neurotransmitters in the brain. Which neurotransmitter is primarily targeted by
significant emotional distress, he engages in excessive joking and laughter when antipsychotic medications?
discussing his past experiences. Which defense mechanism is the client employing? A. Serotonin – target by antidepressants
A. Reaction formation B. Dopamine – to lower dopamine
B. Intellectualization C. Norepinephrine – target by antidepressants and anxiolytics
C. Displacement D. Acetylcholine – involve in various mental processes
D. Humor. 87. A nurse is assessing a client who has been prescribed an antidepressant
82. A 25-year-old female client, who recently failed an important exam, becomes overly medication. The nurse explains to the client that the medication primarily works by
invested in a new hobby to distract herself from feelings of disappointment and failure. increasing the levels of which neurotransmitter in the brain?
Which defense mechanism is the client employing? A. Serotonin.
A. Sublimation – direct her emotions into a more effective outlet B. Dopamine
B. Regression C. Norepinephrine
C. Repression D. GABA
D. Identification 88. A nurse is caring for a patient who has been prescribed a mood stabilizing
83. A 35-year-old male client, who is facing financial difficulties, constantly talks about medication for the management of bipolar disorder. The nurse explains to the patient
how wealthy and successful he will be in the future. He refuses to acknowledge his that the medication primarily works by:
A. Increasing dopamine levels in the brain. B. Confront Mr. S directly and ask him to allow others to speak.
B. Inhibiting the reuptake of serotonin. C. Ignore Mr. S's interruptions and redirect the conversation to another client.
C. Enhancing the effects of gamma-aminobutyric acid (GABA). D. Assign a group leader role to Mr. S to channel his energy into facilitating the
D. Blocking the receptors for norepinephrine. group.
89. A nurse is educating a patient who has been prescribed an anxiolytic medication 94. Nurse J is providing care for a client with bipolar disorder who is experiencing a
for the management of anxiety disorder. The nurse explains that the medication manic episode. The client is engaging in impulsive and reckless behavior. Which
primarily works by: nursing intervention is the highest priority?
A. Increasing serotonin levels in the brain. A. Setting limits and boundaries for the client's behavior.
B. Enhancing the effects of gamma-aminobutyric acid (GABA). B. Administering prescribed medications as ordered.
C. Blocking the reuptake of norepinephrine. C. Providing education on the importance of sleep hygiene.
D. Inhibiting the release of dopamine. D. Collaborating with the client on developing a safety plan.
90. A nurse is caring for a patient who has been prescribed an anticholinergic 95. Nurse G is caring for a client with major depressive disorder who has been
medication for the management of extrapyramidal symptoms (EPS) caused by experiencing feelings of guilt and worthlessness. The client states, "I'm such a burden
antipsychotic medication. The nurse explains that the anticholinergic medication to my family. They would be better off without me." Which therapeutic response by
primarily works by: Nurse G is most appropriate?
A. Increasing dopamine levels in the brain. A. "You shouldn't think like that. Your family loves you."
B. Blocking acetylcholine receptors in the brain. B. "I understand how you feel, but remember that your family needs you."
C. Enhancing the effects of serotonin in the brain. – antidepressants C. "Why do you feel like a burden? Can you give me some examples?"
D. Inhibiting the reuptake of norepinephrine. – antidepressants D. "Everything will get better soon, just hang in there."
91. Nurse M is conducting a therapeutic communication session with a client who has 96. A nurse is conducting an initial assessment on a client who has been admitted to
been diagnosed with depression. The client expresses feelings of hopelessness and the psychiatric unit. The nurse notices that the client's body language is tense, with
lack of motivation. Nurse M responds by saying, "I understand how you feel. It must clenched fists and a frowning facial expression. The client speaks in a loud voice and
be really difficult for you." Which statement best describes Nurse M's response? maintains a piercing gaze. The nurse interprets these behaviors as indicative of:
A. Assertive communication A. Assertive communication – clear, direst and respectful communication ofone’s
B. Active listening. though and feelings
C. Nonverbal communication B. Nonverbal communication – refers to the body language, facial expression and
D. Nontherapeutic communication the eye contact
92. Nurse R is interacting with a client with schizophrenia who is experiencing auditory C. Covert cues – hidden or indirect messages that are not easily observable
hallucinations. The client repeatedly states, "They are telling me to hurt myself." Which D. Overt cues – visible or obvious messages and easily observable
therapeutic communication technique should Nurse R utilize? 97. During a therapy session, a client expresses feelings of sadness and states, "I feel
A. Reflecting. like things will never get better." The nurse responds, "I understand how you feel. It
B. Offering self must be really difficult for you right now." This response by the nurse is an example of:
C. Giving advice A. Active observation
D. Providing reassurance B. Congruent message
93. Nurse L is conducting a group therapy session for clients with substance use C. Nontherapeutic communication – false reassurance
disorders. One of the clients, Mr. S, frequently interrupts others and dominates the D. Directive role
conversation. What is the most appropriate nursing intervention to address this 98. A nurse is providing therapeutic communication to a client who is experiencing
behavior? anxiety. The nurse maintains eye contact, nods occasionally, and uses verbal cues
A. Encourage other group members to speak up and express their feelings. such as "I see" and "Tell me more." These actions by the nurse are examples of:
A. Active listening.
B. Abstract messages – messages that are unclear or lacking specific details
C. Closed body positions – physical postures that indicates lack of openness and
defensive
D. Assertive communication
99. A nurse is conducting a group therapy session for clients with substance use
disorders. One client frequently interrupts others and dominates the conversation,
making it difficult for other group members to participate. The nurse addresses this
behavior by saying, "I appreciate your enthusiasm, but I would like to give others a
chance to share their thoughts as well." This response by the nurse is an example of:
A. Directive role – provides guidance or directions to the client
B. Therapeutic communication.
C. Concrete message – specific and clear messages that leaves little room for
interpretation
D. Nonverbal communication
100. A nurse is providing therapeutic communication to a client who is experiencing
grief after the loss of a loved one. The nurse says, "I understand how you feel. I lost
someone close to me too, and it was really tough." This response by the nurse is an
example of:
A. Empathy
B. Nontherapeutic communication.
C. Nondirective role
D. Intimate zone

You might also like